January 2014

Page 1

Independent News on Advances in Hematology/Oncology CLINICALONCOLOGY.COM • January 2014 • Vol. 9, No. 1

INSIDE SOLID TUMORS Safe to omit radiation in elderly breast cancer patients ....................................... 9 USPSTF recommends lung cancer screening for high-risk patients ................. 10

HEMATOLOGIC DISEASE Study suggests wider use of half-match transplant ................................... 3

CURRENT PRACTICE Farhad Ravandi, MD: How I manage newly diagnosed acute promyelocytic leukemia ..................................... 4

REPORT Addressing Misconceptions About Selective Internal Radiation Therapy See insert after page 12 2

The Heterogeneity of Cancer Offers Targeted Therapy Opportunities See page 6

IMAGES in ONCOLOGY

2013 ASH

CAR T-Cell Therapies: Positive Early Results New Orleans—Of the many emerging therapeutic options discussed at the 2013 annual meeting of the American Society of Hematology (ASH), perhaps none have generated more impressive clinical activity than chimeric antigen receptor (CAR)-expressing T-cell therapy. Numerous small studies testing the gene-engineered therapy in highly challenging patients were presented. Repeatedly, the results showed complete and relatively durable remissions in patients who had exhausted practically all other lines of therapy. see CAR T CELL, page 8

“Untitled,” a squamous carcinoma cell masquerading as a kaleidoscopic tadpole. For more information see page 2.

SABCS 2013

Vogl, NY...

A Less Toxic Therapy for LowRisk Breast Cancer

SABCS: Examining NSABP B32

San Antonio—A new study has demonstrated that treatment with adjuvant paclitaxel and trastuzumab (Herceptin, Genentech) is highly effective in patients with stage I, node-negative, HER2-positive breast cancer. Although this finding comes from a Phase II trial, clinicians believe it will have a great influence on practice. “Paclitaxel and trastuzumab can be considered a reasonable and appealing approach for the majority of patients with stage I HER2-positive breast cancer,” said Sara Tolaney, MPH, MD, a medical oncologist from Dana-Farber Cancer Institute in Boston. “Standard see LESS TOXIC, C page 10

Trial should have asked ‘How big is the detriment from occult nodal metastases?’

T

homas Julian, MD, associate director of the Allegheny Breast Center in Pittsburgh, presented the 10-year results of NSABP B32 at the San Antonio Breast Cancer Symposium on Dec. 11 (abstract S2-05). He correctly concluded that sentinel node biopsy, when skillfully done, is adequate treatment of the axilla if the node is negative on routine histologic examination, with no need for a completion axillary dissection. This was a randomized study of about 5,600 women designed to Steven Vogl, MD examine this point, and I believe it excluded a substantial benefit from axillary dissection in this population. In B32, women who had negative sentinel nodes on routine exam (N=3,989) then had the same nodes subjected to immunohistochemistry (IHC) on two more slices. see VOGL, NY, Y page 16

RE VIE WS & COMMENTAR IES

Expert Insights From Massachusetts General Hospital Cancer Center Afatinib extends PFS in patients with EGFR lung cancer ................ 12 Christopher G. Azzoli, MD

Site-specific benefit of bevacizumab in metastatic colorectal cancer ........................ 14 Janet E. Murphy, MD, MPH


2

CLINICAL ONCOLOGY NEWS

CLINICAL ONCOLOGY NEWS • JANUARY 2014 • CLINICALONCOLOGY.COM

EDITORIAL BOARD

Solid Tumors Bone Metastases Allan Lipton, MD Milton S. Hershey Medical Center, Penn State University Hershey, PA

Breast Cancer

Prostate Cancer

Charles F. von Gunten, MD

Michael A. Carducci, MD Johns Hopkins Kimmel Cancer Center Baltimore, MD

Memorial Sloan-Kettering Cancer Center, Weill Cornell Medical College New York, NY

Oncology Nursing

Hematologic Malignancies Dana-Farber Cancer Institute, Harvard Medical School Boston, MA

Memorial Sloan-Kettering Cancer Center, Weill Cornell Medical College New York, NY

Edward Chu, MD University of Pittsburgh Cancer Institute, University of Pittsburgh Pittsburgh, PA

Mayo Clinic Rochester, MN

Mary Lou Bowers, MBA The Pritchard Group Rockville, MD

Syed A. Abutalib, MD

Leonard Saltz, MD

Cancer Treatment Centers of America Zion, Illinois

Cindy O’Bryant, PharmD

Matt Brow

University of Colorado Cancer Center Denver, CO

VP, Public Policy & Reimbursement Strategy McKesson Specialty Health The US Oncology Network Washington, DC

Sara S. Kim, PharmD

Richard Stone, MD

University of Texas, MD Anderson Cancer Center Houston, TX

The Mount Sinai Medical Center New York, NY

Infection Control Susan K. Seo, MD Memorial Sloan-Kettering Cancer Center New York, NY

On the Cover

O

ur cover features the artwork of Marie Sicari, MD, a pathologist, visual artist and performer whose digital art photography project focuses on imagery of human disease. Dr. Sicari specializes in the fields of anatomic pathology and dermatopathology. Dr. Sicari’s digital abstract art explores the fantastic imagery of microscopic pathology as a potential tool to access and explore the mind-body-spirit connection. Her work reflects the paradigm shift occurring in medicine toward the inclusion of holistic approaches and the role of creative arts in the healing process. If you are interested in purchasing this piece or other work from her collection, Dr. Sicari may be reached at www.behance.net/MarieSicari

Community Oncology John W. Finnie, MD Mercy Medical Center St. Louis, MO

Ephraim Casper, MD Memorial Sloan-Kettering Cancer Center, Weill Cornell Medical College New York, NY

Policy and Management

Pharmacy

Cathy Eng, MD

Gastrointestinal Cancer and Sarcoma

Cleveland Clinic Foundation Lerner College of Medicine of Case Western Reserve University Cleveland, OH

University of Alabama Birmingham, AL

Dana-Farber Cancer Institute, Harvard Medical School Boston, MA

Memorial Sloan-Kettering Cancer Center, Weill Cornell Medical College New York, NY

Paul J. Ford, PhD

City of Hope National Medical Center Duarte, CA

University of Texas, MD Anderson Cancer Center Houston, TX

Shaji Kumar, MD

Gastrointestinal Cancer

Betty Ferrell, RN, PhD

Michele Neskey, MMSc, PA-C

Harry Erba, MD, PhD Maura N. Dickler, MD

Joseph P. DeMarco, PhD Cleveland State University Cleveland, OH

Jennifer R. Brown, MD, PhD Andrew Seidman, MD

Bioethics

University of California, San Diego, CA

Michael J. Fisch, MD, MPH University of Texas, MD Anderson Cancer Center Houston, TX

Genitourinary y Cancer Ronald M. Bukowski, MD Taussig Cancer Center, Cleveland Clinic Foundation Cleveland, OH

Gynecologic y g Cancer Maurie Markman, MD Cancer Treatment Centers of America Philadelphia, PA

Steven Vogl, MD Medical Oncologist New York, NY

Mission Statement Symptom Control and Palliative Care

T

he mission of Clinical Oncology News is to be an independent source of unbiased, accurate and reliable news combined with in-depth expert analysis about the issues that oncologists and hematologists care about most. We strive to be a valuable source for oncologists and hematologists in providing the best possible care for their patients.

William S. Breitbart, MD Memorial Sloan-Kettering Cancer Center New York, NY

Lung g and Head and Neck Cancers Edward S. Kim, MD

Steven D. Passik, PhD

Levine Cancer Institute, Carolinas HealthCare Charlotte, NC

Vanderbilt University Medical Center Nashville, TN

Editorial Philosophy The Editorial Board of Clinical Oncology News is instrumental in guiding the content that appears in the magazine. A significant proportion of the news coverage comes from studies presented at cancer conventions and meetings. Prior to these meetings such as the ASCO annual meeting, board members are asked to identify abstracts that should be covered in their area of specialty. Board members, in their area of specialty, are also consulted about review article topics, and whether or not to cover specific trends, studies that appear in peer-reviewed journals, reports from government agencies, etc., and review the articles before they go to print. Additionally, all news articles that appear in Clinical Oncology Newss are sent to the sources quoted in each article to review and verify the accuracy of the article’s content.

Lung g Cancer,, Emesis Richard J. Gralla, MD Albert Einstein College of Medicine, New York, NY

Joseph V. Pergolizzi Jr., MD Johns Hopkins University School of Medicine Baltimore, MD

Russell K. Portenoy, MD Beth Israel Medical Center New York, NY

Educational review articles, commentaries, and other clinician-authored pieces are written exclusively by the named authors.


HEMATOLOGIC DISEASE

CLINICAL ONCOLOGY NEWS • JANUARY 2014 • CLINICALONCOLOGY.COM

ASH 2013

Study Suggests Wider Use of Half-Match Transplant Immediate implications for transplantation referral New Orleans—New data on reducedintensity, related-donor haploidentical bone marrow transplant (haploBMT) suggests that patients in their 60s and even 70s can have outcomes comparable to those in patients in their 50s, a result that already has altered transplant protocol at Johns Hopkins University, where the study was conducted (abstracts 2091 and 158). The data was presented at the 2013 annual meeting of the American Society of Hematology (ASH). Reduced-intensity haploBMT is offered in a relatively limited number of centers, but recent advances in technique have increased the safety of the procedure substantially, extending the option of transplant to more cancer patients. “The practice-changing implications of these data are twofold. Number 1 is that based on the advances in how they are performed, haploidentical, related transplants are a very reasonable consideration for patients who do not have matched donors and are in desperate need of a transplant. Number 2 is that based on these data, we recently lifted our age limit for reduced-intensity transplants at Johns Hopkins,” said lead author Yvette L. Kasamon, MD, of the Sidney Kimmel Comprehensive Cancer Center at Johns Hopkins University in Baltimore. HaploBMT is potentially curative, but safety outcomes have been inferior to those achieved with HLA-matched donors. The advantage of haploBMT is that nearly all patients have at least one HLA-haploidentical relative and also can get the transplant quickly. Only one in four siblings will be HLA-identical. Despite expanding bone marrow donor registries, currently unrelated matches can be found for only about 50% of patients, and an unrelated donor search usually takes months, delaying therapy. Among advances that have improved the outcomes of haploBMT, the most significant one may be the use of high-dose post-transplant cyclophosphamide to

Dosing scheme for reduced intensity, related haploidentical bone marrow transplant with high-dose post-transplantation cyclophosphamide (from the Blood and Marrow Transplant Clinical Trials Network). T-cell replete BMT Cy 14.5 mg/kg/day

TBI 200 cGy

BMT, Day -6

-5

-4

Clinical Oncology News (ISSN 1933-0677) is published monthly by McMahon Publishing, 545 West 45th Street, New York, NY 10036. Corp. Office, 83 Peaceable Street, Redding CT 06896 Copyright© 2014 McMahon Publishing, New York, NY. All rights reserved.

-3

-2

-1

Fludarabine

0

5

Cy 50 mg/kg/day

BMT, bone marrow transplant; Cy, cyclophosphamide; MMF, mycophenolate mofetil; TBI, total body irradiation.

reduce the risk for graft-versus-host disease (GVHD) and graft rejection. In the protocol at Johns Hopkins, which is performed on an outpatient basis, conditioning consists of fludarabine (Fludara, Sanofi), cyclophosphamide and 200 cGy total body irradiation. Beginning three days after transplant, high-dose cyclophosphamide (50 mg/kg per day) is administered on two consecutive days. Tacrolimus and mycophenolate mofetil (CellCept, Roche) are started five days after transplant. “At Hopkins, if you have a relative who is a potential haplo donor, we no longer in most cases even look for a matched unrelated donor,” Dr. Kasamon said. More than 400 haplo transplants have been performed at Johns Hopkins. Age-related outcomes of reducedintensity, related haploBMT with posttransplant cyclophosphamide in 273 patients, aged 50 to 75 at the time of BMT, were presented at the ASH meeting. With a median follow-up of 2.1 years, the two-year progression-free survival (PFS) estimates were indistinguishable among age groups: 39% for 119 patients aged 50 to 59; 36% for 127 patients aged

60 to 69; and 39% for 27 patients aged 70 to 75 ((P=0.9). The two-year overall survival estimates also were similar (56% for those aged 60-69, 51% for those aged 50-59, and 44% for those aged 70-75; P=0.9), as was the risk for non-relapse mortality. The overall risk for severe acute GVHD was only 3%. “We used to limit reduced-intensity transplant to age 75, which explains why this was the upper limit in this analysis,” Dr. Kasamon said. “Now, we look at performance status and overall functioning. For sure, there are patients 75 years old who are in better shape than some patients who are 40 or 50 years of age.” In fact, compared with age 50 to 59, age 60 and older did not emerge as a significant factor in either univariate or multivariate analyses of PFS. In many hematologic malignancies, hematopoietic stem cell transplantation offers the best opportunity for sustained disease control and cure. HaploBMT is one of several initiatives to expand eligibility for these procedures. Due to improvements in preventing GVHD and graft rejection, this data suggests older

age may no longer be a barrier to success. Thomas R. Spitzer, MD, director of Bone Marrow Transplantation at Massachusetts General Hospital in Boston, said clinicians should be paying attention to the wider use of haploBMT. For the right candidate, he considers the Johns Hopkins haploBMT strategy to be a “revolutionary approach” to transplantation for patients without a matched donor in their family. Many institutions in the United States, including his own, are turning to this technique with promising efficacy and good safety results, even as specific indications for the approach are still being defined. Although “its ultimate role in the field will be determined by prospective randomized trials comparing this approach to other forms of stem cell transplantation,” Dr. Spitzer suggested that this does appear to be a strategy to increase the pool of candidates for BMT.

POSTMASTER: Please send address changes to Clinical Oncology News, 545 W. 45th St., 8th Floor, New York, NY 10036. www.mcmahonmed.com

SALES STAFF

Brandy Wilson, Circulation Coordinator

Julianna Dawson, Publication Director jdawson@mcmahonmed.com

Mark Neufeld, Associate Director, Project Management

Lauren Smith, Classified Advertising Sales lauren@mcmahonmed.com

MCMAHON PUBLISHING

ART AND PRODUCTION STAFF

Raymond E. McMahon, Publisher & CEO, Managing Partner

Michele McMahon Velle, Creative Director, MAX Graphics

Van Velle, President, Partner

®

McMahon Publishing is a 42-year-old, family-owned medical publishing and medical education company. McMahon publishes seven clinical newspapers, seven special editions, and continuing medical education and custom publications.

MMF, Tacrolimus

EDITORIAL STAFF Kevin Horty, Group Publication Editor khorty@mcmahonmed.com Gabriel Miller, Managing Editor gmiller@mcmahonmed.com James Prudden, Group Editorial Director Robin B. Weisberg, Manager, Editorial Services Elizabeth Zhong, Associate Copy Chief

Frank Tagarello, Senior Art Director/ Managing Director, MAX Graphics Dan Radebaugh, Director of Production and Technical Operations

—Ted Bosworth Drs. Kasamon and Spitzer reported no relevant conflicts of interest.

Matthew McMahon, General Manager, Partner Lauren Smith, Michael McMahon, Michele McMahon Velle, Rosanne C. McMahon, Partners

3


4

CURRENT PRACTICE

CLINICAL ONCOLOGY NEWS • JANUARY 2014 • CLINICALONCOLOGY.COM

How I Manage ...

Newly Diagnosed Acute Promyelocytic Leukemia

A

cute promyelocytic leukemia (APL) is a disease that demonstrates how progress in understanding cancer biology can be translated successfully into immense improvement in patient outcomes. The disease, which accounts for about 10% of cases of acute myeloid leukemia (AML), has long been recognized as a distinct type of AML with the characteristic granules and multiple Auer rods in the classical type (accounting for 80% of cases of APL) being generally well recognized by trained pathologists. Other features that help with early recognition include a characteristic immunophenotype, younger age at diagnosis, a low white blood cell (WBC) count at presentation and a strong association with coagulopathy. However, about 20% of patients have the microgranular variant and a definitive diagnosis requires establishing molecular evidence of the disease by showing the presence of the pathognomonic translocation between chromosomes 15 and 17 and/or the resulting fusion transcript, PML-RARa. This is possible using standard karyotype analysis, fluorescent in situ hybridization or polymerase chain reaction (PCR). Rare cases may have alternative partners for the RARa gene, which is important due to their insensitivity to all-trans retinoic acid (ATRA). A rapid diagnostic test utilizing anti-PML antibodies can be used to increase the speed of diagnosis.

Why should ATRA be started at first suspicion of APL? Early initiation of therapy with ATRA is of paramount importance at first suspicion of APL to prevent catastrophic bleeding, that is, pulmonary and intracranial hemorrhages and early mortality (approximately 10%). This is a wellrecognized and accepted management strategy in patients whose clinical presentation and peripheral smear film is consistent with APL. Aggressive correction of coagulopathy through the use of blood products including platelet

transfusion to maintain a platelet count greater than 50×109/L; fresh frozen plasma to correct coagulation factor deficiency and maintain a close to normal prothrombin time and partial thromboplastin time; and cryoprecipitate to maintain fibrinogen greater than 150 mg/dL, is another important aspect of disease management. Other important aspects include detection and management of infections with appropriate IV antibiotics; close attention to patients’ fluid status including avoiding excessive fluids while maintaining adequate hydration; and prevention and treatment

of tumor lysis syndrome using fluids, allopurinol or other agents, as necessary.

In the modern era, should we continue to incorporate conventional chemotherapeutic agents into the treatment of patients newly diagnosed with APL? Historically, induction therapy included the combination of ATRA with idarubicin with or without cytarabine. The European APL Group has suggested that cytarabine may be an important component of therapy (potentially reducing the risk for rare central

AT A GLANCE • Early initiation of therapy with ATRA is of paramount importance at the first suspicion of APL. • ATRA and ATO have been associated with the development of potentially fatal differentiation syndrome. • The North American Intergroup demonstrated that adding two cycles of ATO after achieving CR significantly improves RFS. • It is of paramount importance to establish complete molecular remission using a sensitive PCR test at the end of consolidation. • If a patient’s PCR test becomes positive, it should be repeated using bone marrow within two to four weeks, and if positive again, salvage therapy should be initiated. • The role of maintenance therapy at the end of consolidation continues to evolve. • The treatment of APL during pregnancy can be difficult, as chemotherapy, ATRA and ATO are all considered teratogenic.

Farhad Ravandi, MD Professor of Medicine Department of Leukemia University of Texas MD Anderson Cancer Center Houston, Texas

nervous system relapses), however, several studies have demonstrated that regimens combining ATRA and idarubicin are as effective and probably less toxic. More recently, a number of studies have incorporated arsenic trioxide (ATO) into upfront treatment. The North American Intergroup demonstrated that adding two cycles of ATO after achieving complete remission (CR) significantly improved relapsefree survival (RFS). A group from India and one from Iran also have reported high responses with single-agent ATO. However, it appears that, at least for high-risk disease, ATO alone may not be sufficient for the long-term disease elimination. Investigators from China demonstrated that combining ATO with ATRA for induction was associated with a significantly higher reduction in disease burden (as measured by real-time, quantitative PCR). At the University of Texas MD Anderson Cancer Center, we have advocated a “chemotherapy-free” regimen of ATRA plus ATO with the use of gemtuzumab ozogamicin (GO) in patients with high-risk disease (defined as WBC at presentation >10×109/L) and those patients with lowrisk disease whose WBC rises above 10×109/L during the course of therapy. Long-term follow-up of these patients shows that this regimen is not only very effective in achieving near-universal molecular remission, but that relapses also are very uncommon. These data led to a randomized study conducted by Italian and German groups where the


CURRENT PRACTICE

CLINICAL ONCOLOGY NEWS • JANUARY 2014 • CLINICALONCOLOGY.COM

ATRA plus ATO regimen was compared with ATRA plus idarubicin in patients with low-risk APL (WBC ≤10×109/L, which accounts for about two-thirds of patients in most series). The anthracycline-free regimen was associated with a significant improvement in event-free and overall survival and, in my opinion, was clearly established as the standard of care at least in patients with lowrisk disease. In this study, hydroxyurea was used to control rising WBC, which is important because GO recently has been withdrawn from the U.S. market. Another option is the use of a single dose of idarubicin in patients with highrisk disease and those with rising WBC.

When should “differentiation syndrome” be suspected in patients with APL? ATRA and ATO have been associated with the development of potentially fatal differentiation syndrome, although the risk for this complication does not appear to be increased in the combination regimen. Differentiation syndrome should be suspected in any patient receiving these agents who develops fever, shortness of breath, cough, pulmonary infiltrates on a chest x-ray or edema. It is a serious complication that is best managed by withdrawing the responsible agents and initiating therapy with dexamethasone. Some groups, including ours, use prophylactic doses of steroids at onset of therapy, but it is important to remember that this strategy does not eliminate the risk for differentiation syndrome and one should remain wary of this possibility.

How do I stratify risk in patients with APL? Historically, the risk stratification classification proposed by Miguel Sanz, MD, PhD, of Hospital Universitario La Fe in Valencia, Spain, and colleagues divides patients with APL into three groups: low-, intermediate- and highrisk depending not only on their WBC at diagnosis but also their platelet count (above and below 40×109/L). However, many groups, including ours, believe that the outcome of low- and intermediate-risk patients is very similar (especially using the ATRA plus ATO regimen) and therefore many centers only define risk by WBC at presentation, as described above.

What is the role of monitoring minimal residual disease in patients with APL? An important aspect of treating patients with APL is monitoring for minimal residual disease (MRD) because

initiate therapy with ATRA alone, deliver the fetus as soon as possible and complete consolidation with appropriate measures such as anthracyclines and/ or ATO. The use of ATO at any stage of pregnancy should be avoided, as it has shown to be highly toxic to the embryo in animal studies.

Further Reading

Several abnormal promyelocytes with intense azurophilic granulation and Auer rods.

APL is the only subtype of AML where persistence or recurrence of MRD as measured by PCR has been unequivocally associated with relapse. It is, however, important to note that a large proportion of patients remain PCR-positive at the end of induction with the achievement of morphologic CR, and this does not portend a worse outcome (and may be the result of terminally differentiated leukemia cells that have not yet undergone apoptosis). It is, however, of paramount importance to establish molecular CR using a sensitive PCR test at the end of consolidation and to initiate further therapy for the rare patient who remains PCR-positive at the end of consolidation despite adequate therapy.

How do I monitor for MRD in APL? The risk for relapse for low-risk patients who have completed adequate consolidation is so low that many experts question the necessity of monitoring for MRD with PCR beyond completion of therapy. However, for high-risk patients, most experts would recommend monitoring with a sensitive PCR test in the peripheral blood every three months for the first two years, and perhaps less frequently thereafter. Although bone marrow PCR is probably more sensitive and can potentially detect molecular relapse about one month before peripheral blood, most experts consider monitoring using peripheral blood samples adequate.

How does MRD positivity influence treatment decisions in patients with APL? If a patient’s PCR test becomes positive, it should be repeated from bone marrow within two to four weeks, and if positive again, salvage therapy should be initiated. A patient who achieves a second molecular remission after salvage treatment may be considered for an autologous stem cell transplant, although the best approach in this

setting remains undefined due to a lack of extensive data. Patients who remain PCR-positive despite adequate salvage therapy should be considered for allogeneic transplantation.

What is the role of maintenance therapy in patients with APL?

1. Swerdlow SH, Campo E, Harris NL, et al. (eds). World Health Organization Classification of Tumours of Haematopoietic and Lymphoid Tissues. Lyon, France: IARC Press, 2008. 2. Powell BL, Moser B, Stock W, et al. Arsenic trioxide improves event-free and overall survival for adults with acute promyelocytic leukemia: North American Leukemia Intergroup Study C9710. Blood. 2010;116:37513577, PMID: 20705755. 3. Lo-Coco F, Avvisati G, Vignetti M, et al. Retinoic acid and arsenic trioxide for acute promyelocytic leukemia. N Engl J Med. 2013;369:111-121, PMID: 23841729.

The role of maintenance therapy at the end of consolidation is also evolving. With the ATRA plus chemotherapy regimens, one year of maintenance with or without oral chemotherapy (in the form of mercaptopurine and methotrexate) and intermittent ATRA has been advocated in most regimens. However, the ATRA plus ATO regimen does not include any maintenance at the end of the eight-month consolidation period and it can be argued that with the availability of very sensitive PCR tests for monitoring MRD and the availability of effective salvage for the few patients who relapse, the role of maintenance therapy will be further questioned in the future.

4. Mathews V, George B, Chendamarai E, et al. Single-agent arsenic trioxide in the treatment of newly diagnosed acute promyelocytic leukemia: long-term followup data. J Clin Oncol. 2010;28:3866-3871, PMID:20644086.

How do you counsel patients with APL during pregnancy?

8. Cardonick E, Iacobucci A. Use of chemotherapy during human pregnancy. Lancet Oncol. 2004;5:283-291, PMID:15120665.

The treatment of APL during pregnancy can be difficult, as chemotherapy, ATRA and ATO are all considered to be teratogenic. In particular, in the first trimester of pregnancy, the options for a successful outcome for the fetus are very limited and termination of the pregnancy should be strongly considered and followed by rapid initiation of therapy for the mother. It has been suggested that daunorubicin (which is less lipophilic and therefore potentially associated with decreased placental transfer) is the preferred anthracycline. In the cases of second- and third-trimester pregnancy, treatment with ATRA and anthracyclines is considered reasonably safe, although the latter may be associated with increased risk for abortion, low birth weight, premature delivery and neonatal sepsis. Therefore, one potential possibility is to

5. Ghavamzadeh A, Alimoghaddam K, Rostami S, et al. Phase II study of single-agent arsenic trioxide for the front-line therapy of acute promyelocytic leukemia. J Clin Oncol. 2011;29:2753-2757, PMID:21646615. 6. Sanz MA, Martín G, González M, et al. Riskadapted treatment of acute promyelocytic leukemia with all-trans-retinoic acid and anthracycline monochemotherapy: a multicenter study by the PETHEMA group. Blood. 2004;103:1237-1243, PMID:14576047. 7.

Ravandi F, Estey E, Jones D, et al. Effective treatment of acute promyelocytic leukemia with all-trans-retinoic acid, arsenic trioxide, and gemtuzumab ozogamicin. J Clin Oncol. 2009;27:504-510, PMID: 19075265.

Series Editor Syed Abutalib, MD Assistant Director, Hematology & Stem Cell Transplantation Program Cancer Treatment Centers of America, Zion, Illinois

Coming Soon How I Manage: Waldenström Macroglobulinemia by Morie Gertz, MD Mayo Clinic

5


THE SCIENCE BEHIND POSITIVE PATIENT OUTCOMES

The Heterogeneity of Cancer Offers Targeted Therapy Opportunities Faculty Andrea Califano, PhD Chair, Columbia University Department of Systems Biology Director, JP Sulzberger Columbia Genome Center Associate Director, NCI-Designated Herbert Irving Comprehensive Cancer Center Clyde and Helen Wu Professor of Chemical Systems Biology Columbia University and NewYork-Presbyterian/Columbia University Medical Center New York, New York

Mark A. Rubin, MD Director, The Institute for Precision Medicine Vice Chair for Experimental Pathology Director, Translational Research Laboratory Services Homer T. Hirst III Professor of Oncology Weill Cornell Medical College and NewYork-Presbyterian/Weill Cornell Medical Center New York, New York

A

t NewYork-Presbyterian Hospital, initiatives to tailor cancer therapy to the specific genomic and molecular features of a patient’s tumor are advancing rapidly in the clinic. The Institute for Precision Medicine was created by Weill Cornell Medical College and NewYork-Presbyterian/ Weill Cornell Medical Center specifically with this mandate in mind. At NewYork-Presbyterian/Columbia University Medical Center and elsewhere on the Columbia University campus, meanwhile, pioneering work characterizing the interplay of genetic characteristics and molecular events in cancer cell pathology is providing a new path to therapies for challenging cancers in trial-eligible patients. “We are trying to understand cancer by looking at the regulatory networks within the cell,” explained Andrea Califano, PhD, chair, Columbia University Department of Systems Biology. “Integrative network-based association studies [INAS] are for the first time allowing us to separate the handful of driver events from the thousands of transcriptional, posttranscriptional, and metabolic events that could be potentially linked to a given cancer. This is an important departure from genome-wide association studies [GWAS], which look linearly at the germline DNA for common variants associated with cancer growth.” Dr. Califano emphasized, “Cell regulation is not generally a linear phenomenon.”

Targeting Mutations Individualized cancer care, foreseen for a decade or more, is already underway, and the NewYork-Presbyterian initiatives represent bold next steps. In the Precision Medicine Clinic of the Institute for Precision Medicine, whole-exome

6

CLINICAL ONCOLOGY NEWS • JANUARY 2014

genome sequencing is being performed for patients with microarrays, and expression of cloned DNA—to isolate the advanced cancer to identify targetable mutations. The events unique to the growth of a cancer type. This ongoing immediate effect is the opportunity to identify one of a work has already allowed Dr. Rubin to publish findings on an growing number of targetable mutations to which treat- array of novel biomarkers for aggressive forms of prostate ment can be directed. At the same time, this sequencing will and other cancers. generate its own library of recurring mutations for which a “We have started sequencing patients with advanced new generation of targeted therapies might be developed. bladder, prostate, lung, and brain cancers, and are per“A patient’s genome can now be sequenced in a mat- forming real-time analyses that we hope will guide us in ter of days at an acceptable cost,” said Mark A. Rubin, MD, understanding events that confer or inhibit susceptibility director of the Institute for Precision Medicine. “This has to highly targeted therapies,” Dr. Rubin said. These princibeen a key advance with far-reaching implications.” This ples extend to many other cancer types, and the Institute not only means identifying and developing therapies for for Precision Medicine provides the framework for expandevents responsible for the development of malignancy, but ing this approach. also developing therapies for the genetic and molecular This is not theoretical. Dr. Rubin noted that the work events that permit a tumor to regrow when initial treat- with sequencing in advanced prostate cancer has already ments lose efficacy. It is reasonable to expect both of these revealed actionable information that may provide opportutypes of therapies to be targetable. nities to improve outcome. The challenge will be to develop “Next-generation sequencing will allow us to look for clinical trials around these novel discoveries. As data genermechanisms of growth when treatments fail, creating sec- ated by serial sequencing of advanced cancers accrue, the ond opportunities to understand and inhibit tumor growth,” opportunity for the Institute for Precision Medicine to fulfill explained Dr. Rubin, who is also director of Translational its mission increases. These data will capture key genetic and Research Laboratory Services at NewYork-Presbyterian/Weill molecular drivers of cancer from which those most relevant Cornell. He co-led the team that characterized the fusion of to an individual patient can be identified and targeted. TMPRSS2 and ETS transcription factors, which is now recognized as one of the earliest events of prostate cancer Regulatory Network Models invasion. More recent work has defined the landscape of The heterogeneity of cancer underlies the efforts to better prostate cancer mutations. individualize care on both campuses of NewYork-Presbyterian The work at the Institute for Precision Medicine will not Hospital. Within cancer types further stratified by histology, only address cancer but also other diseases, such as genetic, stage, or other broad characteristics, mechanisms of growth cardiovascular, and neurodegenerative disorders, for which can differ, which explains the variability in response to thergenomic sequencing may provide new insights into pre- apy. Information gained from sequencing studies suggests vention and treatment. In many ways, the advanced work in that therapy needs to be further individualized even among understanding the signals that initiate abnormal cell growth patients whose cancer has emerged from the same genetic in cancer is informing research in other diseases. The efforts pathways. This is because additional heterogeneity in drivto control tumor growth by identifying and then suppress- ers of growth is produced by a complex interaction between ing the underlying molecular signals have major implica- culprit genes and the molecular events that follow. Drilling tions for understanding a full spectrum of disease states. As down to key and targetable drivers of growth in individual opposed to cytotoxic therapies designed to eradicate can- patients is the focus of the work at NewYork-Presbyterian/ cer cells, the inhibition of cancer growth Columbia and Columbia University, where involves understanding its biolDr. Califano is evaluating regulatory A large ogy at a fundamental level. networks through INAS. analysis of tissue samples The hope is that, through “Regulatory network increased understandmodels attempt to from patients with neuroendocrine ing, better and more understand the celtumors of the gastrointestinal tract was precise therapies can lular machinery that conducted specifically to develop an integrative be initiated. connects the genetic In advance of the mutations in a cancer network-based association study capable founding of the Institute patient with the abnorof isolating targetable drivers of for Precision Medicine, Dr. mal behavior of their cells. malignancy. Rubin participated in and led These models take into considthe types of translational research eration the influence of genetics and teams on which these advances depend. It epigenetics to identify the unique vulnerabiliis complex work that requires collaboration between clini- ties of the cancer cell,” explained Dr. Califano, who also serves cians, pathologists, computational biologists, and basic sci- as the associate director of the NCI-designated Herbert Irving entists. The data are collated from multiple strategies—such Comprehensive Cancer Center at NewYork-Presbyterian/ as laser capture microdissection, high-throughput tissue Columbia. He suggested a parallel to INAS might be


continue to produce new factors to consider, and appear to be important in the context of genetic susceptibility as well as in the microenvironment of the cell at risk. Yet, within this overwhelming number of factors with potential relevance to cancer development, the final pathways to oncogenesis in the individual patient appear to be much smaller, at least in some cancers. As an example, Dr. Califano noted that there are dozens of genetic alterations associated with glioma but a much more limited number of molecular subtypes. In cancers like these, inhibition of the final common pathway, or cancer bottleneck, may benefit a large subset of patients with very different mutations. The opportunity for benefit in both cases depends on improved characterization of cancer—which appears to be possible with INAS—and then individualization of therapy. The concept of individualized therapy is already well established in a few malignancies. For example, tissue profiling is performed routinely in patients with non-small cell lung cancer because of the relevance of specific biological markers to drug selection. However, as the signaling systems and mechanisms of cancer are better underFigure. A Columbia Systems Biology study, led by Dennis Vitkup, PhD, identified metabolic stood, new targets for treatment will be identified and expression changes in cancer as well as hundreds of potential drug targets. Up-regulation is in the role of individualized therapy will expand. The work red, down-regulation is in blue. underway at NewYork-Presbyterian Hospital is at the foreNature Biotechnology. 2013;31:522-529. front of this effort. “The foundation of the Institute for Precision Medicine is conceptualized as accessing the blueprint of a complex elec- downstream effects, the focus from INAS on likely targeta- an exciting step,” Dr. Rubin said. “We expect to contribute tronic circuit with millions of components to understand why ble drivers of cancer can be narrowed. substantially to the rapid advances being made in identifyit is broken and how to fix it. Rather than addressing oncogening cell signals that participate in human diseases, not just esis as a linear phenomenon stemming from a single gene Cancer Bottlenecks cancer. The Institute will allow us to readily translate some mutation, INAS understands cancer as a product of multiple, “What we are looking for are the critical bottlenecks in the of the advances into improved patient care and, we hope, complex, and nonlinear events dependent on multiple fac- complex mechanics of cancer that are essential to tumor sur- better outcomes.” tors and creating unique vulnerabilities, which can be tar- vival and can be targeted pharmacologically,” said Dr. Califano. Dr. Califano made a similar point. As the interplay between geted pharmacologically. As noted above, this is not just theoretical. A large anal- factors leading to malignancy are better understood, it has This approach circumvents the limitations of GWAS. ysis of tissue samples from patients with neuroendocrine become clear that tumors that appear virtually identical are Although GWAS has identified mutations closely associ- tumors of the gastrointestinal tract was conducted spe- driven by very different signaling mechanisms within the ated with increased cancer risk, not all patients with a spe- cifically to develop an INAS capable of isolatcell. Targeted therapies that are effective for cific mutation develop cancer or develop cancer via the exact ing targetable drivers of malignancy. one individual may therefore not have same set of underlying mechanisms. For instance, therapies The work allows simultaneous, similar efficacy in another. As a that are highly effective in a patient with a specific driver interactive signaling sysresult, cancer therapy is likely As the signaling systems and mutation may fail completely in another patient with the tems to be considered, to become increasingly same exact mutation. The complexity of the mechanics of which is likely to be individualized as more mechanisms of cancer are better cancer is further underlined by the fact that genomic altera- more representative of details about key steps understood, new targets for treatment will tions in individual tumor subclones are continuously evolv- cell biology than linear in the signaling process be identified and the role of individualized ing and being selected for, even once the malignancy is step-by-step signaling are isolated. established. This explains loss of response to treatment over pathways. “The regulatory nettherapy will expand. time and emphasizes the need for individualized treatment. “Pathway models are easwork model provides a Although the changes in activated and deactivated ier to conceptualize and lend framework to sort out the key genes, protein expression, and other molecular events themselves more easily to validation targets in complex processes depenpotentially associated with cancer in any single patient when a change in the pathway leads to a change dent on multiple factors,” Dr. Califano said. “In are vast, Dr. Califano pointed out that they are not ran- in cell function. However, individual transcription factors may most cases, sustained control of cancer is not likely to be dom. Rather, the promotion of malignancy can be traced regulate hundreds of thousands of cell context-dependent achieved by suppressing the activity of a single gene but back to a handful of driver genes, or master regulators of targets for which ultimate function is determined by syn- will require a combination of strategies against multiple cancer, that produce specific behavior. These genes are ergistic interactions between 2 or more genes,” Dr. Califano pathways. We think a better understanding of the cellular frequently not the ones that are mutated but are those explained. This complexity may explain why single pathways machinery will expand the opportunities to adapt treatdownstream from the mutated genes. By attempting to have proven incomplete for understanding cancer biology. ment to the specific features of the malignancy we are tryreconstruct gene-regulatory networks and understand the He noted that efforts to better characterize oncogenesis ing to control.”

Copyright © 2014, McMahon Publishing, 545 West 45th Street, New York, NY 10036. Printed in the USA. All rights reserved, including the right of reproduction, in whole or in part, in any form. BB1315

Image courtesy of Dennis Vitkup, PhD

Supported by

CLINICAL ONCOLOGY NEWS • JANUARY 2014

7


8

HEMATOLOGIC DISEASE

CLINICAL ONCOLOGY NEWS • JANUARY 2014 • CLINICALONCOLOGY.COM

CAR T CELL continued from page 1

For the brief time this strategy has been in clinical testing, “we are seeing some fantastic outcomes,” said Marco Davila, MD, PhD, a leukemia specialist at Memorial Sloan-Kettering Cancer Center (MSKCC) in New York City. He presented data on a series of 16 patients with B-cell acute lymphoblastic leukemia (B-ALL) who received CAR T cells. Other data were presented on the treatment of chronic lymphocytic leukemia (CLL) and acute myeloid leukemia (AML). The construction of the CAR T cells used in these studies differs, but the principle is the same. A form of adoptive immunotherapy, CARs join the antigen-binding domain of an antibody with the signaling domain of a T-cell receptor. T cells with the engineered CAR are specifically targeted and cytotoxic to the tumor cells that express that antigen. For the CAR therapy developed at MSKCC to treat B-ALL, T cells were first isolated from the patients scheduled to receive this therapy and then genetically modified for targeting the CD19 surface protein. This involved fusing the signaling domains of the CD28 co-stimulatory receptor and the zeta chain of the CD3 complex with antigenbinding domains of an anti-CD19 antibody. The 19-28z CAR T cells created by this technique, which can be engineered in less than two weeks, are then returned to the patient. “CD19 antigens are expressed on all normal and malignant B cells,” Dr. Davila said. “Expression of the 19-28z CAR by a T cell promotes binding of the CD19 antigen and triggers cytotoxicity, cytokine release and proliferation upon engagement of CD19.” Other CAR T cells, including those used for B-ALL, are constructed differently. For example, the same signaling domain was fused with the antigen-binding domains from a different anti-CD19 antibody to create the FMC63-28Z CAR T cell tested against B-ALL at the National Cancer Institute (NCI). Yet another CAR targeting CD19, modified with the 41BB co-stimulatory protein instead of CD28, was used against B-ALL in studies performed at the University of Pennsylvania. Promising data on all three CARs were presented at the 2013 ASH annual meeting. The data presented on B-ALL by Dr. Davila was generally representative.

‘These results show, for the first time, that small numbers of donor-derived allogeneic anti-CD19 CAR T cells can cause regression of highly treatmentresistant B-cell malignancies after allogeneic HSCT without causing graft-versus-host disease.’ —James Kochenderfer, MD Part of an experience with more than 60 patients treated at MSKCC with CAR T cells, all 16 of the B-ALL patients in this series had been pretreated heavily. Most of them had detectable disease at the time of CAR infusion. Of the patients, 88% achieved a complete remission, which is very high given the advanced stage of the disease. Furthermore, most of these were not only complete responses, but complete molecular remissions (CMR), an indicator of even greater disease control. Dr. Davila recounted two representative cases in which CMR was attained within two weeks of the infusion, even though the bone marrow blast percentage before therapy exceeded 60%. Two months after therapy, both patients recovered complete bone marrow function with no evidence of disease. Seven of the 16 patients in the series overall have undergone stem cell transplant. The side effects of CAR T-cell therapy include fever, malaise, hypotension and hypoxia, which appear to be related to cytokine release. This explains the correlation between the intensity of these side effects and tumor burden at the time

of treatment. Notably, the only relapses observed so far occurred in patients who received steroids to manage these adverse events. Subsequent use of tocilizumab (Actemra, Hoffman-La Roche), an interleukin-6 inhibitor, reduced these adverse events without exerting any apparent effect on outcomes. The feasibility of CAR therapy in patients with advanced disease and multiple previous therapies, including hematopoietic stem cell transplant (HSCT), was underscored in a series of 10 patients with a broader range of advanced cancers treated at the NCI. Although only three of these patients achieved a substantial regression of malignancy, it was noted that T cells could be collected, engineered for CAR and reinfused even in patients with multiple previous transplants. One such woman has remained in remission for 12 months after the CAR T-cell infusion despite three previous transplants. “These results show, for the first time, that small numbers of donorderived allogeneic anti-CD19 CAR T cells can cause regression of highly treatment-resistant B-cell malignancies

after allogeneic HSCT without causing graft-versus-host disease,” said James Kochenderfer, MD, an investigator in the Experimental Transplantation and Immunology Branch of the NCI. In a separate study, he also reported six complete remissions with anti-CD19 CAR therapy in a series of 15 patients with refractory large B-cell lymphomas. It was identified as the first report of benefit with CAR therapy in this group of patients. All of the studies presented at ASH were Phase I, so CAR T-cell therapy remains highly experimental. Noting that responses have not been as great in indolent lymphomas, Dr. Davila speculated that some forms of leukemia, such as B-ALL, may be better suited to this approach than others. For patients with highly advanced, refractory B-ALL, Dr. Davila indicated that referral to a clinical trial with CAR T-cell therapy is likely to become increasingly attractive if benefits are sustained. —Ted Bosworth Drs. Davila and Kochenderfer reported no relevant conflicts of interest.

Send us your news Clinical Oncology News appreciates news tips and suggestions for coverage from readers.

Write to managing editor Gabriel Miller at gmiller@mcmahonmed.com


SOLID TUMORS

CLINICAL ONCOLOGY NEWS • JANUARY 2014 • CLINICALONCOLOGY.COM

SABCS 2013

Safe To Omit Radiation in Older Breast Cancer Patients San Antonio—A randomized study of 1,300 patients has shown that omitting radiation is a reasonable option for women older than age 65 with early-stage, hormone receptor–positive breast cancer who undergo breast-conserving surgery and endocrine therapy. Omission of radiation in this patient group had no effect on survival and only a 2.8% increase in the risk for ipsilateral breast tumor recurrence (IBTR) over five years. “Although radiation therapy reduces IBTR significantly, the absolute difference is very small,” said Ian Kunkler, FRCR, a professor of clinical oncology at the Edinburgh Cancer Research Center in Scotland. “An increasing proportion of patients that we see in the clinic, over 50%, have a relatively benign natural history. Radiotherapy might represent overtreatment.” Dr. Kunkler presented the study at the recent San Antonio Breast Cancer Symposium (SABCS; abstract S2-01). The study included patients 65 years or older who had breast-conserving surgery for hormone receptor–positive, invasive breast cancer, and then were treated with adjuvant endocrine therapy. Eligibility criteria included tumor size up to 3 cm, excision margins of at least 1 mm on histologic assessment and no axillary node involvement on histologic assessment. Patients could not have previous in situ or invasive carcinoma of either breast, or current or previous malignancy within the past five years, except for non-melanomatous skin cancer or carcinoma in situ of the cervix. Patients received either whole breast irradiation (40-50 Gy in 15-25 fractions; n=658) or no irradiation (n=668). Baseline characteristics were well balanced between treatment groups. Overall, the five-year actuarial local control rate was 4.1% in patients who did not receive radiation and 1.3% in patients who received radiation ((P=0.002). There was no difference in overall survival. Factors such as tumor size, tumor grade, margin size and patient age (65-69 or >70) did not affect outcomes. The investigators also conducted an unplanned analysis of local recurrence according to estrogen receptor (ER) status. High ER was defined as ER-positive, ER 7 or greater, fmol greater than 20, staining greater than 20 and triplepositive status. In the high-ER group, the addition of radiotherapy caused only a small reduction (2.4%) in IBTR (3.2% vs. 0.8%). In the low-ER group, 11.1% of patients in the nonirradiated group had local recurrence compared with 0% in the radiation group. “Omission of postoperative radiation therapy in hormone-rich patients, based on the five-year rate of IBTR and overall survival, appears to be a reasonable

option,” Dr. Kunkler said. In 2005, the National Comprehensive Cancer Network amended its clinical practice guidelines to reflect that omitting radiation is a reasonable alternative in older patients with ER-positive, clinically node-negative T1 tumors who receive hormone therapy, however, clinicians have been slow to adopt this practice. The data presented at SABCS may

swing the pendulum to less radiation. “This study gives me a little more confidence that omitting radiation might be the safe thing to do in the right patient,” said Kent Osborne, MD, director of the Dan L. Duncan Cancer Center and the Lester and Sue Smith Breast Center at Baylor College of Medicine in Houston. “The absolute benefit [of radiation] is smaller, and I think that may well be

enough to change practice. The small number of patients who do recur in the breast in the no-radiation group can be salvaged at that time by re-lumpectomy plus radiation or mastectomy.” —Kate O’Rourke Drs. Kunkler and Osborne reported no relevant disclosures.

RESEARCH Advancing Genomic Medicine To Create Precision Therapies.

Targeted for Personalized Medicine nyp.org 877 NYP-WELL (877-697-9355)

9


10

SOLID TUMORS

CLINICAL ONCOLOGY NEWS • JANUARY 2014 • CLINICALONCOLOGY.COM

LESS TOXIC continued from page 1

regimens from pivotal trials can be considered for patients with particularly high-risk features.” Dr. Tolaney presented the study at the recent San Antonio Breast Cancer Symposium (SABCS; abstract S1-04). Although clinical trials have demonstrated that trastuzumab plus chemotherapy cuts breast cancer recurrence risk in half and improves overall survival, it is unclear how it performs in lowrisk patients. Few patients in the clinical trials that tested trastuzumab plus chemotherapy had stage I, HER2-positive breast cancer, and virtually no patients had tumors that were 1 cm or smaller. Because the absolute benefits of a cancer therapy are likely to be smaller in patients who fall into a low-risk group, but carry the same toxicity burden, researchers have been keen to

Table. Size of Primary Tumor Tumor Classification, cm

Prevalence, %

T1a ≤0.5

19

T1b >0.5 to ≤1.0

31

T1c >1.0 to ≤2.0

42

T2 >2.0 to ≤3.0

9

‘For people who are waiting for evidence, this is about as much as we are going to get. The tactic that [the researchers] chose was “if we do a big enough nonrandomized study and we show that the risk for relapse is very low, we will convince the world that under these conditions, the patients do very well.” That is the study you saw.’ —Clifford Hudis, MD identify a less toxic regimen. Launching a Phase III trial to identify such a regimen, however, seems near to impossible. Recruiting to an arm without trastuzumab would be challenging, due to accumulating evidence of the recurrence risk in these patients and preferences by both patients and physicians for an anti-HER2–based therapy. Enrolling a trastuzumab-only arm would have been difficult, given the data in the metastatic setting suggesting far greater activity when chemotherapy is added to this agent. For these reasons, researchers launched a Phase II trial of trastuzumab plus the less toxic chemotherapy paclitaxel in approximately 400 patients with HER2-positive, node-negative breast cancer with tumors smaller than 3 cm. Patients received paclitaxel 80 mg/m2 plus trastuzumab 2 mg/kg for 12 weeks, followed by 13 every-three-week doses of trastuzumab 6 mg/kg. Radiation and hormone therapy were started after completion of paclitaxel. Roughly 67% of patients were estrogen receptor-positive; 50% of tumors were 1 cm or smaller; and 42% were between 1 and 2 cm (Table).

The three-year, disease-free survival (DFS) rate was 98.7%, and the three-year, recurrence-free interval was 99.2%. Only 10 DFS events were identified in a total of 406 patients: two distant recurrences; four invasive locoregional recurrences; three contralateral HER2-positive cancers; and one death from ovarian cancer. The most common grade 3/4 adverse events were neutropenia in just under 5% of patients, neuropathy in 3% and fatigue in 2%. The most common adverse events of any grade were fatigue (22%), diarrhea (13%), neuropathy (13%), neutropenia (11%) and hyperglycemia (10%). Two patients developed symptomatic congestive heart failure, both of whom recovered their left ventricular ejection fraction after discontinuing trastuzumab. According to Dr. Tolaney, although an all-biologic regimen might have an even lower toxicity and should be pursued, adding other biologic agents to the docetaxel–trastuzumab backbone is unlikely to add substantial benefit in this patient population, given the excellent outcomes seen in this trial. According to Clifford Hudis, MD, the chief of the Breast Cancer Medicine Service at Memorial Sloan-Kettering Cancer

USPSTF Recommends Lung Cancer Screening For High-Risk Patients

O

n December 30, the U.S. Preventive Services Task Force (USPSTF) issued a final recommendation statement advocating annual lung cancer screening for individuals at high risk. This statement departs from the task force’s 2004 recommendation, which found insufficient evidence for or against lung cancer screening. According to the new recommendation, screening should use low-dose computed tomography (CT), and those screened should be healthy with a 30 pack-year or greater history of smoking,

who have smoked within the past 15 years, and who are aged 55 to 79 years. A pack-year means one pack of cigarettes per day for one year. The recommendation is based on a systematic review of the evidence, particularly the National Lung Screening Trial, which is the largest randomized controlled trial on the subject to date, according to the recommendation statement. “This is finally a step in the right direction for lung cancer screening,” said Edward S. Kim, MD, who was not involved in developing the statement.

“It’s important to realize our most recent data has demonstrated that CT screening in people greater than 55 years old who have at least a 30 pack-year smoking history can decrease overall mortality by 20%.” Dr. Kim, who is chair of Solid Tumor Oncology and Investigational Therapeutics at the Levine Cancer Institute in Charlotte, N.C., added, “It’s my hope that CT scans for the appropriate individuals will be covered by the payers, thus allowing people to obtain the appropriate necessary screening.”

Center in New York City, the study is likely to be the most practice-changing research presented at SABCS this year. Clinicians, he said, have been wondering whether they can avoid multi-agent chemotherapy regimens in the low-risk breast cancer population described in the study. “For people who are waiting for evidence, this is about as much as we are going to get,” he said. “The tactic that they [the researchers] chose was ‘if we do a big enough nonrandomized study and we show that the risk for relapse is very low, we will convince the world that under these conditions, the patients do very well.’ That is the study you saw.” Dr. Hudis said clinicians should consider the regimen for stage I, HER2-positive, node-negative breast cancer patients. “This chemotherapy regimen is shorter and much less disruptive for patients, and generally, all around easier,” he said. “For patients with small tumors, only rarely will they experience relapse with this regimen.” –Kate O’Rourke Dr. Tolaney disclosed research funding from Genentech. Dr. Hudis had no relevant disclosures.

According to the task force’s categories of suggestions for practice, the new recommendation is grade B, which, like grade A, means the service should be offered or provided. Addressing the absence of oncologists on the USPSTF, Dr. Kim said, “I do believe if there are going to be practical recommendations made about a certain disease type, one would think that having members who are experienced in the field of study—in this case that would be pulmonary, surgical or medical oncology in lung cancer—would be helpful.” —George Ochoa Dr. Kim reported no relevant financial conflicts of interest.

Clinical Oncology News TWEETS! Follow us @ClinOncNews And send your Twitter handle to managing editor Gabriel Miller at gmiller@mcmahonmed.com so we can follow you.


Harnessing the Immune System in NSCLC Implications of Emerging Data and Immunotherapeutic Strategies for Personalized Medicine To participate in this FREE CME activity, log on to

Release date: October 1, 2013

www.CMEZone.com

Expiration date: September 30, 2014

Editor

TARGET AUDIENCE

Suresh S. Ramalingam, MD

The target audience for this activity is medical oncologists, hematology/oncology fellows, oncology specialty pharmacists, and other health care professionals involved in the management of individuals with nonsmall cell lung cancer (NSCLC).

Professor of Hematology and Medical Oncology Director, Division of Medical Oncology Emory University School of Medicine Winship Cancer Institute Atlanta, Georgia

EDUCATIONAL OBJECTIVES At the conclusion of this activity, participants should be able to:

Faculty Julie R. Brahmer, MD

1 Review fundamental concepts of antitumor immune responses in NSCLC.

Associate Professor Johns Hopkins University School of Medicine Baltimore, Maryland

2 Evaluate key efficacy and safety data from ongoing clinical trials evaluating immunotherapeutic strategies for NSCLC, including tecemotide (formerly known as L-BLP25), belagenpumatucel-L, melanoma-associated antigenA3 (MAGE-A3) vaccine, immune checkpoint inhibitors, toll-like receptor agonists, and mycobacterial adjuvant-based agents.

John Nemunaitis, MD Executive Medical Director Mary Crowley Cancer Research Centers Dallas, Texas

Roman Perez-Soler, MD Professor of Medicine Chair, Department of Oncology Montefiore Einstein Center for Cancer Care Chief, Division of Medical Oncology Department of Medicine Deputy Director Albert Einstein Cancer Center Bronx, New York

3 Identify effective immunotherapeutic strategies for early- and advanced-stage NSCLC based on patient and disease characteristics. 4 Recall the ongoing clinical trials evaluating immunotherapeutic approaches for NSCLC to aid appropriate patients for study participation.

MEDIA Monograph

ESTIMATED TIME TO COMPLETE ACTIVITY 1.0 hour

Sponsored by

DESIGNATION OF CREDIT PHYSICIAN CONTINUING EDUCATION Accreditation Statement Educational Concepts Group, LLC is accredited by the Accreditation Council for Continuing Medical

Supported by an Educational Grant from

Education to provide continuing medical education for physicians.

Credit Designation Statement Educational Concepts Group, LLC designates this enduring material for a maximum of 1.0 AMA PRA Category 1 Credit™. ™ Physicians should claim only the credit commensurate with the extent of their participation in the activity.

PHARMACIST CONTINUING EDUCATION Educational Concepts Group, LLC is accredited by the Accreditation Council for Pharmacy Education as a provider of continuing pharmacy education. Educational Concepts Group, LLC designates this continuing education activity for 1.0 contact hour (0.10 CEU) (UAN 0199-0000-13-034-H01-P).

TYPE OF ACTIVITY Knowledge-based

METHOD OF PARTICIPATION There are no fees for participating and receiving CME/CE credit for this activity. During the period October 1, 2013 through September 30, 2014, participants must 1) read the educational objectives and faculty disclosures; 2) study the educational activity; and 3) complete the post-activity assessment.

CME/CE CREDIT Physicians and pharmacists who complete the postactivity assessment with a score of 70% or better may view and print their credit letter or statement of credit via the website, www.educationalconcepts.net.

Distributed via


12

REVIEWS & COMMENTARIES

CLINICAL ONCOLOGY NEWS • JANUARY 2014 • CLINICALONCOLOGY.COM

Expert Insights From Massachusetts General Hospital Cancer Center Each month, Clinical Oncology News summarizes findings from several recently published, important studies and then asks clinicians from a top cancer center to offer their perspectives. The cancer center providing the expert commentaries changes every three months. Cancer centers are chosen based on reputation, availability and regional diversity, and we seek to have a mix of academic institutions together with regionally important hospitals with expertise in cancer care. Massachusetts General Hospital Cancer Center continues its commentaries, both here and on our website at clinicaloncology.com. We hope you find this Reviews & Commentaries section to be a valuable tool.

Afatinib Extends PFS in Patients With EGFR Lung Cancer From Journal of Clinical Oncology

P

reliminary findings of the LUXLung 3 study indicate that treatment with afatinib for advanced lung adenocarcinoma patients with epidermal growth factor receptor (EGFR) mutations is superior to standard chemotherapy. In this global, open-label, Phase III, randomized study, eligible patients with stage IIIB/IV lung adenocarcinoma were screened for activating mutations in EGFR. Afatinib (Gilotrif, Boehringer Ingelheim) is an oral medication that blocks signals from EGFR. Researchers hypothesized that genotype-specific

treatment for these patients would produce results that exceed standard chemotherapy. Of 1,269 eligible patients screened, 452 were found to have the mutation, and 345 were eventually placed into one of two treatment arms. In the first group, 230 patients were given oral afatinib, 40 mg daily, and in the second, 115 patients received standardized six-cycle treatment of IV cisplatin (75 mg/m2) and pemetrexed (500 mg/m2) once every 21 days. Lecia V. Sequist, MD, of Massachusetts General Hospital, Boston, and her international colleagues found that median progression-free survival (PFS) was

EXPERT INSIGHT Christopher G. Azzoli, MD Hematology/Oncology Department of Medicine Massachusetts General Hospital Cancer Center Boston

W

ith each new drug comes new hope for patients with advanced lung cancer. The most important thing about this multinational study by Dr. Sequist and her colleagues is that it led the FDA to approve afatinib, an oral, irreversible inhibitor of EGFR and HER2. Oncologists now have another powerful weapon for their patients in the fight against EGFR-mutant lung cancer, a disease that afflicts tens of thousands of Americans per year. Like its sister drugs, erlotinib and gefitinib, afatinib is better than traditional, platinum-based chemotherapy in treating patients with advanced non-small cell lung cancer

with activating/sensitizing mutations in EGFR. Dr. Sequist’s study teaches us that afatinib, at a daily dose of 40 mg, causes diarrhea, skin rash, fingernail changes and mouth sores, which require close attention in the first few weeks of therapy. These AEs are reversible and treatable, and a lower dose of afatinib may be helpful in selected patients to improve tolerability and allow continuous therapy. As for efficacy, Dr. Sequist’s study showed a median PFS of more than 13 months for first-line afatinib in patients with EGFR exon 19 deletion or L858R. This is the best-reported result to date for first-line use of an EGFR

superior for the afatinib group. Independent oncologists and radiologists who were blinded to treatment options evaluated computed tomography or magnetic resonance imaging assessments regularly. Their evaluations showed PFS of 11.1 months for the afatinib group versus 6.9 months for the standard chemotherapy group (hazard ratio [HR], 0.58; 95% confidence interval [CI], 0.43-0.78; P=0.001). Median PFS for patients with exon 19 deletions and L858R EGFR mutations (n=308) was 13.6 months for the afatinib treatment group and 6.9 months for the standard chemotherapy group (HR, 0.47; 95% CI, 0.34-0.65; P=0.001). Results were published in the Journal

of Clinical Oncologyy (2013;31:3327-3334 PMID: 23816960). Overall survival (OS) data are preliminary, but there appears to be significant delay in the deterioration of symptoms such as worsening of cough and dyspnea in the afatinib group. Adverse events (AEs) were comparable between the two treatment arms, with AEs grade 3 or greater reported by 112 (49%) afatinib patients and by 53 (48%) chemotherapy patients. The authors of the study concluded that afatinib should be considered a treatment option for patients with advanced lung adenocarcinoma who have a known EGFR mutation.

The superiority of EGFR tyrosine kinase inhibitors over platinum-based combination chemotherapy in patients with EGFR mutant lung cancer was first established in 2009. tyrosine kinase inhibitor (TKI) in this setting. Laboratory data shows that afatinib is more potent than gefitinib or erlotinib for inhibiting EGFR signal, and may provide off-target effects that translate into more durable responses. Head-to-head comparisons of afatinib, erlotinib and gefitinib have not been done; however a companion clinical study demonstrated that afatinib has activity in patients previously treated with gefitinib and erlotinib, and may be active against EGFR T790M, the most common resistance mutation to first-generation EGFR TKIs. The superiority of EGFR TKIs over platinum-based combination chemotherapy in patients with EGFR mutant lung cancer was first established in 2009. The past four years have seen

rapid adoption of universal molecular testing of biopsy tissue for advanced non-small cell lung cancer in order to find these special patients who benefit from targeted drugs. This new paradigm has resulted in the rapid discovery of other targets, like ALK and ROS1, and new drugs, like crizotinib and afatinib. Dr. Sequist has been in the thick of these discoveries from the beginning, and has distinguished herself as one of the pioneers leading the charge for this revolution in lung cancer care. Dr. Azzoli reported that he has served as a principal investigator on clinical trials funded by Allos Therapeutics, Genentech and Lilly Oncology, and participated on an advisory board for Boehringer Ingelheim.


REVIEWS & COMMENTARIES

CLINICAL ONCOLOGY NEWS • JANUARY 2014 • CLINICALONCOLOGY.COM

Pomalidomide Plus Dex for Relapsed and Refractory Multiple Myeloma From Lancet Oncology

P

atients with refractory or relapsed and refractory multiple myeloma (MM) may benefit from treatment with a combination of pomalidomide plus low-dose dexamethasone. The international study, led by Jesus San Miguel, MD, PhD, of the University of Salamanca in Spain, followed 455 patients with MM randomly assigned 2:1 to a 28-day cycle of oral pomalidomide (4 mg/day for 21 days) and oral low-dose dexamethasone (40 mg/day on days 1, 8, 15 and 22; n=302) or oral high-dose dexamethasone (40 mg on days 1-4, 9-12 and 17-20; n=153) alone. Investigators recently published their

findings from this Phase III, open-label, randomized trial in Lancet Oncology (2013;14:1055-1066, PMID: 24007748). Patients were eligible if the disease was refractory or relapsed and refractory to standard treatment after multiple attempts. All enrollees had been treated with lenalidomide and bortezomib; patients in both treatment groups averaged five previous treatments. After a median follow-up of 10 months, the progression-free survival (PFS) of patients in the pomalidomide plus low-dose dexamethasone group was 4.0 months (95% confidence interval [CI], 3.6-4.7) and for the high-dose dexamethasone group was 1.9 months (95% CI, 1.9-2.2). The hazard ratio was

0.48 (95% CI, 0.39-0.60; P<0.0001). Treating physicians assessed the response rates. Overall positive response after 10 months was documented in 31% (95 of 302) of patients in the low-dose combination group and in 10% (15 of 153) of those in the highdose dexamethasone group. Complete response rates were demonstrated in 1% (three patients) of the low-dose combination group and absent in the high-dose dexamethasone group. Very good partial response rates were 5% (14 patients) versus less than 1% (one) and partial response rates were 26% (78) versus 9% (14) in the low-dose combination and high-dose dexamethasone treatment groups, respectively.

EXPERT INSIGHT Elizabeth O’Donnell, MD Medical Oncologist, Center for Multiple Myeloma, Massachusetts General Hospital Cancer Center, Boston

B

efore the development of immunomodulatory drugs (IMiDs) and proteasome inhibitors, the prognosis for MM was extremely poor. Over the past 10 years, these drugs have revolutionized the treatment of this disease. Despite these advances, MM remains incurable, and patients ultimately relapse and develop refractory disease. The newest proteasome inhibitor, carfilzomib, was approved in 2012, and a new IMiD has been added to our armamentarium: Pomalidomide, in combination with dexamethasone, is now approved for the treatment of relapsed and refractory disease. In Phase I and II studies of pomalidomide with dexamethasone, activity was demonstrated in patients with MM refractory to bortezomib and lenalidomide.1,2 In this recent Lancet Oncologyy paper, San Miguel and his colleagues published the results of their Phase III trial of pomalidomide in combination with low-dose dexamethasone versus high-dose dexamethasone alone. At the prespecified data cutoff for PFS, the trial

Andrew J. Yee, MD

Noopur S. Raje, MD

Hematology/Oncology, Department of Medicine, Massachusetts General Hospital Cancer Center, Boston

Hematology/Oncology, Department of Medicine, Massachusetts General Hospital Cancer Center, Boston

met the primary end point of PFS and the upper boundary for superior overall survival (OS) was crossed, despite the fact that 45 patients had already crossed over from high-dose dexamethasone to pomalidomide. At that point, patients in the high-dose dexamethasone arm were offered access to pomalidomide. Final OS in the pomalidomide arm was 11.9 versus 7.8 months with dexamethasone alone. Consistent with previous trials, overall response rate was 31% in the pomalidomide plus low-dose steroid group versus 10% in the high-dose dexamethasone group. These are promising results; however, it must be noted that dexamethasone as a single agent is not considered standard of care in the United States in 2013. Taken alone, the response rate of pomalidomide plus low-dose dexamethasone is very good, and the fact that it was durable offers an additional therapy for patients. Another important question is whether or not there were differences in response within the different refractory populations. In the pomalidomide plus

low-dose dexamethasone group, 95% of patients were refractory to lenalidomide, 79% were refractory to bortezomib and 75% were refractory to both lenalidomide and bortezomib. Median PFS was significantly longer with pomalidomide plus low-dose dexamethasone regardless of previous treatment, and PFS was similar in patients refractory to only lenalidomide (3.9 months) compared with patients refractory to both bortezomib and lenalidomide (3.7 months). However, in the final analysis, longer OS was only statistically significant in patients refractory to lenalidomide who received pomalidomide plus low-dose dexamethasone and not in patients refractory to both lenalidomide and bortezomib. Like the other IMiDs, pomalidomide has a favorable side-effect profile. The most common side effects were neutropenia, anemia and thrombocytopenia. Discontinuation of pomalidomide plus low-dose dexamethasone because of AEs was uncommon (9%). Thromboprophylaxis was mandated for patients

Adverse events (AEs) in the two groups were similar. In the low-dose combination treatment group, 68% of 300 patients reported infections and infestations (n=203), 52% reported anemia (n=157) and 51% had neutropenia (n=152). In the high-dose dexamethasone group, AEs affected a majority of patients, with 53% reporting infections and infestations (79 of 150 patients) and 51% reporting anemia (76 of 150 patients). The authors noted a synergistic effect when pomalidomide was combined with dexamethasone as a salvage treatment for patients with MM who are intolerant or refractory to standard therapies.

receiving pomalidomide plus low-dose dexamethasone, although choice was at the discretion of the provider and not specified in the paper. Incidence of deep vein thrombosis and pulmonary embolism was low (<1%). The results of this study of pomalidomide plus low-dose dexamethasone show superior PFS and OS compared with dexamethasone alone. Given its good tolerability, particularly in this somewhat fragile population of patients with relapsed and refractory MM, this regimen can be considered for treatment of relapsed and refractory disease. Trials of pomalidomide in triple-therapy combinations, which include standard anti-myeloma therapies, such as bortezomib and carfilzomib, are ongoing and, in the future, will be considered for relapsed and refractory disease.

References 1. Richardson PG, Siegel D, Baz R, et al. Phase 1 study of pomalidomide MTD, safety, and efficacy in patients with refractory multiple myeloma who have received lenalidomide and bortezomib. Blood. 2013;121:1961-1967, PMID: 23243282. 2. Leleu X, Attal M, Arnulf B, et al. Pomalidomide plus low-dose dexamethasone is active and well tolerated in bortezomib and lenalidomide-refractory multiple myeloma: Intergroupe Francophone du Myelome 2009-02. Blood. 2013;121:19681975, PMID: 23319574.

The authors reported no relevant financial disclosures.

More REVIEWS and COMMENTARIES from Mass General Online Find additional, Web-exclusive expert commentaries on important published studies at

ClinicalOncology.com

13


14

REVIEWS & COMMENTARIES

CLINICAL ONCOLOGY NEWS • JANUARY 2014 • CLINICALONCOLOGY.COM

Site-Specific Benefit of Bevacizumab in Metastatic Colorectal Cancer From Annals of Oncology

A

retrospective study of patients with metastatic colorectal cancer (mCRC) showed improved overall survival (OS) and progression-free survival (PFS) with treatment that included bevacizumab (Avastin, Genentech), compared with the standard first-line therapy of capecitabine (Xeloda, Roche) and oxaliplatin (Eloxatin, Sanofi-aventis; CAPEOX), when the primary tumor site was either the rectum or the sigmoid colon. The researchers hypothesized that the addition of bevacizumab to a course of capecitabine and oxaliplatin (CAPEOXBEV) for patients with mCRC would

target vascular endothelial growth factor A (VEGF-A), and so might be more effective in certain types of cancers. The Scandinavian team, led by M.K. Boisen, MD, gathered data on 880 patients treated for mCRC from oncology centers in Denmark. Included in this group were 213 patients treated between 2003 and 2006, before bevacizumab was adopted as part of standard treatment, and 667 patients who were treated after 2006 with CAPEOXBEV. Patients were further divided into subsets of five primary tumor locations: 1) cecum and ascending colon, 2) right flexure and transverse colon, 3) left flexure and descending colon, 4) sigmoid and rectosigmoid colon, and 5) rectum. Survival status

EXPERT INSIGHT Janet E. Murphy, MD, MPH Hematology/Oncology Department of Medicine Massachusetts General Hospital Cancer Center Boston

A

nti-angiogenic therapy has been standard of care in the first-line treatment of advanced colorectal cancer since 2004, when a landmark Phase III clinical trial demonstrated that the IFL regimen (irinotecan, leucovorin, fluorouracil) combined with bevacizumab (bev) resulted in a nearly five-month survival benefit over IFL alone.1 Recently, two important studies demonstrated the relevance of antiangiogenic therapy continued into the second line, with a significant survival benefit observed with both bev continued past progression2 and zivaflibercept (Zaltrap, Regeneron) with FOLFIRI (leucovorin, fluorouracil, irinotecan) after first progression.3 However, these drugs are expensive and carry the important toxicities of hypertension, hemorrhage, thrombosis and risk for gastrointestinal perforation. As anti-angiogenic therapy plays an expanded role in this disease, identifying predictive biomarkers of benefit and/or resistance is even more crucial. Although ideally we hope to identify which patients benefit most, we must define those patients who derive no benefit at all. Potential predictive biomarkers for bev efficacy include polymorphisms in the VEGF-A gene, plasma VEGF levels, tumor microvessel density and clinical biomarkers such as hypertension

and proteinuria. None has emerged as a gold standard. In this thought-provoking analysis, Boisen et al examine tumor location (which may serve as a proxy for microvessel density and VEGF levels) as a potential predictor of benefit with bev. The authors identify a striking difference in OS benefit, and less so PFS benefit, between proximal and distal tumors treated with CAPEOXBEV. A key strength of the study is its comparator arm: In patients treated with CAPEOX alone, this effect does not hold in the absence of bev. Is this finding clinically relevant? Would it change practice patterns if reproduced prospectively? That is a high bar to meet. Although the authors compare the separation of curves in the bev cohorts with the non–bev-treated cohorts (stratified by tumor location), they do not publish survival statistics in the non–bev-treated group headto-head against the bev-treated arms. Although this omission is methodologically appropriate because it would be hazardous to do so outside of the stringency of a prospective clinical trial, it leaves the reader asking a key question: For proximal tumors, does bev provide less survival benefit or no survival benefit at all? If proximal and distal colon tumors exist on a spectrum of benefit, would we withhold antiangiogenic therapy from patients with

was extracted from a national database, and disease progression notes were obtained from patient records and hospital databases. The authors, publishing in Annals of Oncology (2013;24:2554-2559, PMID: 23864097), found a significant improvement in patients treated with CAPEOXBEV over CAPEOX (median PFS, 9.3 vs. 7.2 months, respectively; hazard ratio [HR], 0.68; 95% confidence interval [CI], 0.56-0.82; median OS, 23.5 vs. 13.0 months; HR, 0.47; 95% CI, 0.380.57), provided the patients had primary tumors of the sigmoid colon or the rectum. This difference persisted after adjustment for possible prognostic factors by multivariate analysis. For

proximal tumors? As anti-angiogenic therapy spans multiple lines of therapy in advanced colorectal cancer, likely not. In contrast, the retrospective analysis of the CRYSTAL study—in which the KRAS mutation was identified as a true biomarker of resistance to treatment, with a clear biological rationale— had implications that were immediately practice-changing.4 Another limitation of this study, identified by the authors, is that molecular features were not considered in their multivariate model. It is well understood that BRAF-mutant colon cancers are predominantly proximal, and prognosis is very poor among patients with metastatic disease in this subgroup—so much so that any small benefit of anti-angiogenic therapy might wash out. Likewise, KRAS mutant tumors, which are emerging as an independent poor prognostic group, are found more proximally, whereas NRAS mutations are more prevalent in distal tumors. This cohort predates molecular study, and data is not available to perform this analysis. This study provokes an interesting question: Although patients with distal tumors appear to benefit more from bev than those with proximal tumors, all patients had metastatic disease. Is the biology of the primary tumors, and their microenvironment, maintained in the metastases, and does this preserve the difference in bev response and OS benefit? This observation merits further study. Ultimately, defining predictive biomarkers of response and resistance to anti-angiogenic therapy will identify patients in whom benefit outweighs toxicity. As anti-VEGF therapy interacts with the tumor vasculature, it should

Microscopic slide of colon cancer cells.

patients treated with CAPEOX, there was no association between primary tumor location and outcome. The authors suggested a possible interaction between primary tumor location and the mechanism of action of bevacizumab. Cancers that originate in the sigmoid colon and the rectum seem more VEGF-dependent and more likely to show an increased response to antiVEGF chemotherapy.

not be affected by driver oncogenes in the tumor itself. By this rationale, it is unlikely we will reach a “benefit/ no benefit” cutpoint in colorectal cancer as seen with KRAS-mutant tumors and cetuximab (Erbitux, Bristol-Myers Squibb). We may ultimately determine that no single biomarker tells the whole story, and that a combination of clinical and molecular predictors will identify responders. While this excellent study suggests that primary tumor location corresponds to a continuum of benefit with bev, in the absence of data that keys in on a subgroup of patients who derive absolutely no benefit from anti-angiogenics, practitioners and patients alike will still favor their use.

References 1. Hurwitz H, Fehrenbacher L, Novotny W, et al. Bevacizumab plus irinotecan, fluorouracil, and leucovorin for metastatic colorectal cancer. N Engl J Med. 2004;350:2335-2342, PMID: 15175435. 2. Bennouna J, Sastre J, Arnold D, et al. Continuation of bevacizumab after first progression in metastatic colorectal cancer (ML18147): a randomised phase 3 trial. Lancet Oncol. 2013;14:29-37, PMID: 23168366. 3. Van Cutsem E, Tabernero J, Lakomy R, et al. Addition of aflibercept to fluorouracil, leucovorin, and irinotecan improves survival in a phase III randomized trial in patients with metastatic colorectal cancer previously treated with an oxaliplatinbased regimen. J Clin Oncol. 2012;30:34993506, PMID: 22949147. 4. Van Cutsem E, Kohne CH, Hitre E, et al. Cetuximab and chemotherapy as initial treatment for metastatic colorectal cancer. N Engl J Med. 2009;360:1408-1417, PMID: 19339720. Dr. Murphy reported no relevant financial disclosures.


SOLID TUMORS

CLINICAL ONCOLOGY NEWS • JANUARY 2014 • CLINICALONCOLOGY.COM

Informing the Mastectomy Versus Lumpectomy Decision Studies show the effect of anxiety on patients’ choice Chicago—Young women with stage I to III breast cancer who have high levels of anxiety are twice as likely to choose a mastectomy over a lumpectomy, according to a new study. The study also identified a low rate of lumpectomy, 38%, in the study population; 24% had a unilateral mastectomy and 38% had a bilateral mastectomy. Lead study author Shoshana Rosenberg, ScD, MPH, a researcher from the Dana-Farber Cancer Institute and Harvard School of Public Health, in Boston, said the results should make physicians pause. “Are women accurately perceiving the risks and is the risk being effectively communicated between the physician and the patient?” she asked. Studies have shown that women with stage I to III breast cancer, regardless of age, have similar survival, whether they undergo a mastectomy or a lumpectomy. In presenting the study at the annual meeting of the American Society of Clinical Oncology (abstract 6507), Dr. Rosenberg pointed out that mastectomy rates are rising in patients between the ages of 18 and 39, but few studies have analyzed factors that may play a role in this increase. In the new study, researchers evaluated factors influencing the decision of 277 women with stage I to III breast cancer who were aged 40 years or younger. These women were part of a multicenter cohort study and resided in Massachusetts and Colorado. Ninety percent had stage I or II disease, 65% had estrogen-positive cancer, and 14% carried the BRCA1 or 2 mutations. Overall, 96% of patients were satisfied with their involvement in the decisionmaking process. In a multivariable analysis, a number of factors were associated with higher rates of mastectomy: BRCA mutation, nodal involvement, HER2 positivity, higher tumor grade, lower body mass index, two or more children, anxiety and a high degree of patient involvement in the decision (Table). Age, marital status, race/ethnicity, tumor estrogen receptor status, having a first-degree relative with breast or ovarian cancer, depression and fear of recurrence were not influencing factors. Anxiety and depression were measured using the Hospital Anxiety and Depression Scale. “We saw a high incidence of mastectomy among young women who had a choice of surgical treatment for their early-stage breast cancer, and we identified some potentially modifiable factors, including patient-driven decision making and anxiety,” Dr. Rosenberg

said. She suggested that interventions aimed at reducing anxiety and fostering fully informed decision making through decision aids may be useful. Sarah Hawley, PhD, MPH, an expert in patient–physician education at the University of Michigan, Ann Arbor VA Medical Center, agreed. “There are lots of decision aids out there addressing various different types of cancer

decisions, from screening to survivorship, and I think that, in general, we can conclude they are effective for improving knowledge about the particular condition or treatment option,” she said. “They may help patients clarify values to prepare for treatment discussions.” Clinicians need to find ways to encourage patients to communicate. “It’s not enough to just educate

[patients] about the options, risks and benefits,” Dr. Hawley said. “We need to give them the tools to bring these issues forward and talk about things they may be uncomfortable bringing up.” —Kate O’Rourke Drs. Rosenberg and Hawley have no relevant disclosures.

®

makes all the difference

With CancerCare, the difference comes from: • Professional oncology social workers • Free counseling • Education and practical help • Up-to-date information • CancerCare for Kids® For needs that go beyond medical care, refer your patients and their loved ones to CancerCare. CancerCare’s free services help people cope with the emotional and practical concerns arising from a cancer diagnosis and are integral to the standard of care for all cancer patients, as recommended by the Institute of Medicine.

Help and Hope

1-800-813-HOPE (4673) www.cancercare.org

15


16

CURRENT PRACTICE

VOGL, NY

CLINICAL ONCOLOGY NEWS • JANUARY 2014 • CLINICALONCOLOGY.COM

Anatomic Staging is Still Important: “Occult” Sentinel Nodal Metastasis in NSABP B32

continued from page 1

EDITORIAL BOARD COMMENTARY

• Occurs in 16% of women with negative sentinel nodes on routine histology.*

Steven Vogl, MD

• Lead to increased risk of systemic relapse.**

Medical Oncologist, New York City

• Increases hazard rate of death by 37%. • Increases hazard rate of recurrence, new primary or death (inverse of DFS) by 27%. • Very rarely leads to local recurrence in the axilla.

The NSABP then sought to answer the question of whether such examination for “occult” nodal metastases is worthwhile by looking at patient outcomes. A short version of the results is that deaths at 10 years increased from 11% to 14.1% and disease-free survival (DFS) events increased from 22.2% to 26.9%, if two extra sections of each sentinel node stained for epithelial cells demonstrated metastatic breast cancer. My assertion is that having even tiny amounts of breast cancer in axillary nodes cannot make the prognosis better. Rather than approach the issue with the question of “Do occult metastases worsen the prognosis?,” I believe the question should have been, “How bad is it to have even minute involvement of axillary nodes?” In other words, this study should have set out to exclude a predefined level of decline in prognosis (distant metasta-

Careful examination of the sentinel axillary nodes should remain available as a technique for clinicians when the increased risk of systemic relapse would change systemic adjuvant therapy: Small HER2-positive and triple-negative cancers in women who could tolerate enhanced systemic therapy, and perhaps big tumors with low and intermediate recurrence scores.*** * And about another 9% if sought by very thin serial sections.2 ** Compared with “truly negative” nodes. *** Or similar results on other genomic prognostic tests.

therapy.1 We do not know how many were HER2-positive, nor was histologic grade centrally reviewed. About 16% of 3,268 patients with negative routine nodal histology on the sentinel nodes had metastases detected by IHC—11% isolated tumor cells (cell collections <0.2 mm in size); 4.4% “micrometastases” (nodal nodules 0.2-2 mm); and 0.4% had “macrometastases” (>2 mm)

sentinel nodes had occult metastases: a 27% reduction in DFS ((P=0.02). This remained significant when adjusted for other prognostic factors, with a 25% reduction in DFS events (P ( =0.01). Isolated tumor cells and micrometastases increased the risk for DFS events similarly (29% and 23%, respectively). DFS was 18% worse with the presence of occult nodal metastases if axil-

Pathologists should subject sentinel axillary nodes to careful examination if the risk for relapse or distant metastases based on known patient characteristics is sufficiently high such that knowledge of the worsened prognosis would change systemic therapy. ses or overall survival) associated with occult nodal involvement. It seems unlikely that we will ever get an even larger randomized trial to answer this noninferiority question with sufficient statistical confidence. We should, therefore, examine the results of B32 carefully to draw what conclusions we can on when, and if, to conduct extraordinarily careful examinations of sentinel nodes by serial sectioning and immunologic staining for small numbers of cancer cells.

B32 Study Results in Detail The population of women in B32 with negative sentinel nodes on routine histology had favorable clinical prognostic characteristics and excellent local and systemic therapy: 84% had primaries less than 2 cm in diameter; 80% of tumors were hormone receptor-positive; 87.5% were treated with lumpectomy; 82% had postoperative irradiation; 75.5% were aged 50 years or older; and 85% had postoperative systemic

that were missed on routine H&E (hematoxylin and eosin) staining of the node. Overall survival at 10 years dropped from 89% to 85.9% if the nodes had occult metastases discovered by more sections and IHC—a 37% increase in the rate of death. This was significant ( =0.03), but when adjusted for oth(P er prognostic factors, the rate of death increased only 26% and the P value was borderline significant ((P=0.06). Isolated tumor cells increased the rate of death by 22% compared with no detectable nodal metastases on IHC, whereas micrometastases (0.2-2 mm) increased the rate of death by 43%. Survival among women with occult metastases in the sentinel node was unaffected by the addition of axillary dissection in this study (the subject of the randomization), with slightly more deaths in the dissection arm (47 vs. 43, of about 300 women in each arm )—a difference that was both trivial and not statistically significant. DFS at 10 years went from 77.8% to 73.1% if the initially negative

lary dissection was added to sentinel node biopsy—a nonsignificant decrease from 76.4% to 69.9% at 10 years ((P=0.2), with a nonsignificant 20% increase in first locoregional recurrences (4.2% to 6.3% at 10 years; P=0.52).

Occult Nodal Disease and Local Control Recurrences in the breast and draining nodes increased from 3.9% to 5.3% if occult axillary nodal involvement was present, a 32% increase that was not significant (P ( =0.17). Axillary recurrences were very rare in the entire study population—0.4% even in those women who did not have an axillary dissection. Among women with occult axillary nodal involvement, first recurrence in the axilla without axillary dissection (n=300) was 1.7%, but went down to 0.6% with axillary dissection (n=316). Most women (87%) in the B32 trial were treated with lumpectomy and breast irradiation that included the lower axilla, in part accounting for the very low axillary recurrence rates.

B32 Nodal Exam Techniques Somewhat Underestimated Occult Nodal Involvement The experimental protocol used in B32 to detect occult disease in the negative sentinel nodes on routine exam included IHC staining on two extra sections. A 54-case subset of B32 patients evaluated exhaustively at the University of Vermont demonstrated that this technique missed many positive nodes with occult disease. In this very small series for which nodes were sliced very thinly to detect collections of isolated tumor cells (ITCs) up to 0.18 mm in size, the frequency of positive blocks increased from 6% to 11.4%, positive nodes from 8% to 12.3%, and positive cases from 17% to 26%.2 If one extends these numbers to the overall B32 study population, the group of women negative for occult metastases (who had only two slices taken from each sentinel node) was likely contaminated by approximately 11% with women whose nodes actually had undetected occult metastases. Had these women, with a likely worse prognosis, been accurately assigned, then the group negative for occult metastases would have done better, and the study might have shown a bigger survival difference in favor of the absence of occult nodal metastases.

Careful Examination of IHC Thin Sections of Sentinel Nodes Will Change Therapy Dr. Julian’s conclusion at SABCS was that careful “routine” IHC examination of the sentinel nodes is not recommended. I fear this will be misinterpreted to mean it should never be done, and hospital histology laboratories will cease to offer this service. Pathologists should subject sentinel axillary nodes to careful examination if the risk for relapse or distant metastases based on known patient characteristics is sufficiently high such that knowledge of the worsened prognosis would change systemic therapy. Control of the axilla is not the issue; systemic relapse and death are the concerns. This assumes the patient could tolerate the systemic therapy that would be given if higher risk is discovered. Based on the B32 results, occult disease in the sentinel node leads to worse outcomes. Knowing the sentinel node has breast cancer in small amounts would be useful in some higher-risk circumstances such as: • The patient is young or middle-aged with a small triple-negative tumor who would otherwise not get adjuvant therapy, but is fit to tolerate it. • The patient has a small HER2-positive tumor. This has a worse prognosis than a small HER2-negative tumor. • The tumor is big (2-5 cm) and hormone-receptor positive with a low or intermediate recurrence score. Until


SOLID TUMORS

CLINICAL ONCOLOGY NEWS • JANUARY 2014 • CLINICALONCOLOGY.COM

Bev Plus Cap ‘Optimal’ for Older mCRC Patients Adding VEGF inhibitor to capecitabine is well tolerated and improves PFS San Francisco—In a rare trial that focused on older patients with metastatic colorectal cancer, the addition of bevacizumab to capecitabine significantly improved progression-free survival (PFS) compared with capecitabine alone. Patients who received the combination therapy had a PFS that reached a mean of 9.1 months—four months longer than patients who received capecitabine alone, representing a 47% decrease in risk for progression (hazard ratio, 0.53, 95% confidence interval, 0.41-0.69; P<0.001). “Based on the efficacy and safety results, bevacizumab plus capecitabine might be an optimal treatment approach to improve outcomes in elderly patients,” said lead author David Cunningham, MD, a consultant medical oncologist and head of the gastrointestinal unit at the Royal Marsden Hospital in London, England. He presented the trial results at the 2013 Gastrointestinal Cancers Symposium (abstract 337). This is the first Phase III trial to prospectively evaluate the use of a biologic in an older patient population. There is critical need for treatments and targeted studies for these patients because the median age of patients diagnosed with colorectal cancer is 69 years, the study’s investigators reported. Older patients are woefully under-represented in clinical trials, with one study suggesting that patients over the age of 65 years account for less than onethird of trial participants ((J Clin Oncol

2003;21:1383-1389, PMID: 12663731). In this open-label trial, known as AVEX, 280 patients with a mean age of 76 years were randomized to receive firstline capecitabine 1,000 mg/m2 daily for two weeks followed by a one-week break or capecitabine in combination with bevacizumab 7.5 mg/kg given every 21 days. Patients had to be at least age 70, have a performance status between 0 and 2 and be deemed unsuitable for combination chemotherapy with irinotecan or oxaliplatin. The study was powered to show a difference in PFS, but not overall survival (OS), between treatment arms. The combination therapy nearly doubled the overall response rate from 10% to 19.3% ((P=0.042). The effect was seen across all recognized subsets, including sex, age, performance status, site of disease and location of the primary tumor. Median OS times were longer in patients who received both agents (20.7 vs. 16.8 months), but the difference was not statistically significant ((P=0.182). Patients remained on bevacizumab therapy for a median of 5.8 months—1.5 months longer than patients who received capecitabine alone. Following the study presentation, several commentators noted that patients’ PFS significantly exceeded the length

of time on treatment. Dr. Cunningham explained that patients were taken off treatment for reasons other than disease progression and were rechallenged if they continued to show a response. Grade 3 or higher adverse events (AEs) occurred more frequently in patients in the combination arm: 59% versus 44.1%. However, grade 5 AEs were more common in the capecitabinealone arm at 11.8% versus 8.2%. AEs associated with bevacizumab included bleeding/hemorrhage (25.4%), hypertension (19.4%), venous thromboembolic events (11.9%), proteinuria (7.5%), arterial thromboembolic events (4.5%), wound healing complications (1.5%), pulmonary hemorrhage/hemoptysis (0.7%) and fistulae (0.7%). “As a clinician treating these patients, I was certainly impressed at how well tolerated this approach was,” said Dr. Cunningham. Robert A. Wolff, MD, professor of gastrointestinal medical oncology at the University of Texas MD Anderson Cancer Center in Houston, said the results are reminiscent of earlier trials that combined bevacizumab with fluorouracil-based chemotherapy. Other studies have shown that improvements in survival are “quite modest” when using

we have the results of the ongoing RxPonder trial, standard therapy for such patients could be considered to include adjuvant chemotherapy. Absent an involved node, many experts now still advise no adjuvant therapy for a HER2-positive primary tumor less than 6 mm in size (Eric Winer, MD, Dana-Farber Cancer Institute, personal communication, December 2013). Nodal involvement makes the prognosis considerably worse for small HER2-positive tumors.3 In an analysis by Manuela Campiglio, PhD, of Italian women with T1 tumors treated with trastuzumab (Herceptin, Genentech) and chemotherapy, the hazard rate for relapse was 3.4 for nodal involvement, 3.1 for negative hormone receptors, and 2.9 for high tumor grade in this patient population. Primary size did not matter, although the number of primaries less than 5 mm was quite small (N=41) in this series.

In B32, the NSABP did monumental work defining the safety and efficacy of sentinel node biopsy for women with breast cancer. If the 37% increase in the risk of death that accompanies occult involvement of the sentinel node would be enough to merit a change in systemic therapy to a regimen that is more effective and whose greater toxicities are now justified by the higher risk for death, then this technique should remain available to surgeons and medical oncologists in every hospital across the country. The costs are small compared with the genetic tests we routinely order and the drugs we routinely administer. If inconvenient at a local level, this testing could be done at referral laboratories, as indeed was done at the University of Vermont in NSABP B32.2 Breast physicians must act now to protect and preserve our ability to very carefully examine sentinel nodes when

we think it will help our patients. If we do not support the value of such testing in print, CMS and the insurance industry may refuse to pay for very careful nodal examination. More annoying, but not as bad, they may force us to divert our efforts from patient care to convincing high-school dropouts, sitting in front of computer screens, that such testing is of value for each patient we think it will help.

‘As a clinician treating these patients, I was certainly impressed at how well tolerated this approach was.’ —David Cunningham, MD

References 1. Krag DN, Anderson SJ, Julian TB, et al. Sentinel-lymph-node resection compared with conventional axillary-lymphnode dissection in clinically node-negative patients with breast cancer: overall survival findings from the NSABP B-32 randomised phase 3 trial. Lancet Oncol. 2010;11:927-933, PMID: 20863759. 2. Weaver DL, Le UP, Dupuis SL, et al. Metastasis detection in sentinel lymph nodes: comparison of a limited widely spaced (NSABP protocol B-32) and a

Colon cancer cells magnified to 2,000 times their size.

more active cytotoxic agents and bevacizumab, he said. On Jan. 23, 2013, the FDA approved bevacizumab for use in combination with fluoropyrimidine-irinotecan– based or fluoropyrimidine-oxaliplatin– based chemotherapy for the treatment of patients with metastatic colorectal cancer whose disease has progressed on a first-line bevacizumab-containing regimen. Bevacizumab is a recombinant humanized monoclonal immunoglobulin G1 antibody that binds to human vascular endothelial growth factor (VEGF), preventing the interaction of VEGF to its receptors on the surface of endothelial cells. —Christina Frangou Dr. Cunningham disclosed that he has received research funding from Amgen, AstraZeneca, Celgene, F. Hoffmann-La Roche Ltd., Merck KGaA and Novartis. The study was funded by AstraZeneca, Merck KGaA and Roche.

comprehensive narrowly spaced paraffin block sectioning strategy. Am J Surg Pathol. 2009;33:1583-1589, PMID: 19730364. 3. Campiglio M, Sandri M, Sasso M, et al. Small HER2-positive breast carcinomas: Prognostic factors to consider in deciding on adjuvant trastuzumab treatment. Presented at: San Antonio Breast Cancer Symposium; December 10-14, 2013; San Antonio, TX. Poster P4-12-06.

What are your thoughts? Clinical Oncology News welcomes letters to the editor. Do you have thoughts on Dr. Vogl’s commentary? Please send comments to gmiller@mcmahonmed.com

17


18

CURRENT PRACTICE

CLINICAL ONCOLOGY NEWS • JANUARY 2014 • CLINICALONCOLOGY.COM

Clinical Conundrums

Prepared by

Syed A. Abutalib, MD

Highlights of Oral Abstracts From ASH 2013, NEJM, JCO and Blood QUESTIONS

1. True or False. A Phase II study pub-

lished in the Journal of Clinical Oncology described a novel strategy of atorvastatin administration in both donors and recipients of matched sibling donor allogeneic hematopoietic stem cell transplantation (allo-HSCT) to prevent acute graftversus-host disease (aGVHD).

2. All of the following are potential

mechanisms of statins in suppressing aGVHD EXCEPT: a. Polarization toward a T-helper type 2 (Th2) cytokine profile b. Suppression of T-cell activation c. Prevention of major histocompatibility complex class II antigen expression on antigen-presenting cells (APCs) d. Polarization toward a Th1 cytokine profile e. Modulation of regulatory T cells leading to their development

3. True or False. A retrospective

study on behalf of the German Hodgkin Study Group described a new prognostic scoring system in older adults (≥60 years) with relapsed/refractory Hodgkin lymphoma.

4. True or False. In patients with

multiple myeloma, investigators from Mayo Clinic in Rochester, Minn., demonstrated comparable long-term outcomes in patients who achieved stringent complete response (sCR) and conventional complete response (CR) undergoing early (<12 months from diagnosis) frontline autologous hematopoietic cell transplantation (auto-HSCT).

5. True or False. After auto-HSCT,

the reconstituting immune landscape may be particularly favorable for breaking immune tolerance through a programmed cell death protein 1 (PD-1)

ANSWERS

1. True.

Atorvastatin administration, in addition to standard GVHD prophylaxis, at a dose of 40 mg per day in healthy sibling donors (14 to 28 days before HSC collection) and recipients (n=30) was not associated with any grade 3/4 adverse events. Cumulative incidence rates of grade II-IV aGVHD at days 100 and 180 were 3.3% (95% confidence interval [CI], 0.2%14.8%) and 11.1% (95% CI, 2.7%-26.4%), respectively; these rates are lower than

10. True

blockade, especially in diffuse large B-cell lymphoma (DLBCL).

or False. Mantle cell lymphoma (MCL): On behalf of the Lymphoma Study Association group, the preliminary results with the RiBVD regimen (rituximab IV, bendamustine IV, bortezomib subcutaneous [Velcade, Millennium], dexamethasone IV) showed high CR rates in older (>65 years) adults with untreated MCL.

6. True

or False. Two studies published in Blood suggest that a myeloablative regimen with cyclophosphamide and IV busulfan (Bu) is associated with better overall survival (OS) than is total body radiation (TBI) and cyclophosphamide in adult Primum non nocere. patients undergoing (First, do no harm.) True or False. allo-HSCT for acute DLBCL: A Phase Ib study myelogenous leukemia (AML). in treatment-naive patients with CD20positive B-cell non-Hodgkin lymphoma True or False. Investigators from (NHL) did not show an acceptable safethe University of Texas MD Ander- ty profile for ibrutinib (Imbruvica, Pharson Cancer Center in Houston report- macyclics) in combination with R-CHOP ed discordant results between measur- (rituximab, cyclophosphamide, doxoruing peripheral blood and bone marrow bicin, vincristine and prednisone). JAK2 p.V617F mutation and allele burden (ratio of mutant allele to total allele) True or False. DLBCL: A subin patients with polycythemia vera (PV), group analysis by sex of the randomessential thrombocytosis (ET) and pri- ized, prospective NHL13 trial providmary myelofibrosis (PM). ed surprising but clear evidence that twice-monthly rituximab maintenance for two years significantly improves ASH 2013 Oral Abstracts: event-free survival (EFS) and progresHighlights—Part I sion-free survival (PFS) among men with DLBCL and grade 3B follicular True or False. Hodgkin lymphoma lymphoma (FL). (HL): In a Phase I/II study, the combination of sirolimus and vorinostat demTrue or False. FL: In the ranonstrated encouraging activity in very domized Phase III SAKK 35/03 trial, heavily pretreated patients with refrac- rituximab maintenance for a maximum tory HL. of five years in patients with FL who received induction with four weekly True or False. Marginal zone lym- rituximab cycles led to improved outphoma (MZL): The final results of a mul- comes compared with short mainteticenter Phase II study showed that the nance rituximab therapy (four rituxcombination of lenalidomide (Revlimid, imab administrations every two months Celgene) and rituximab (R2; Rituxan, for a total of eight months). Genentech) has no activity in patients with MZL compared with other indoTrue or False. FL: Further anallent nonfollicular lymphomas. ysis of the National LymphoCare Study

11.

7.

12.

8.

13.

9.

14.

historical reports in this population with standard prophylactic approaches. Future studies should focus on validating the clinical efficacy of statins in this particular setting.

increased levels of interferon-γγ and tumor necrosis factor-α (Th1 cytokines) after transplantation. Additionally, statins inhibit the APC surface expression of CD40, CD80 and CD86, all of which are important co-stimulatory molecules for T-cell activation.

Hamadani M, Gibson L, Remick S, et al. Sibling donor and recipient immune modulation with atorvastatin for the prophylaxis of acute graft-versus-host disease. J Clin Oncol. 2013;31:4416-4423, PMID: 24166529.

El-Jawahri A, Chen YB. Pleiotropic approach to graft-versus-host disease. J Clin Oncol. 2013;31:4462-4464, PMID: 24166516.

2. D.

3. True. A prognostic score incorpo-

Atorvastatin causes higher expression of interleukin-10, a Th2 cytokine, and placebo treatment showed

rating three risk factors—early relapse (<12 months), clinical stage III/IV and

Assistant Director Hematology & Stem Cell Transplantation Program Cancer Treatment Centers of America Zion, Illinois

identified high lactate dehydrogenase, poor Eastern Cooperative Oncology Group (ECOG) performance status, B symptoms, and bone marrow involvement as risk factors in patients with FL who are likely to progress within two years of R-CHOP therapy.

15.

True or False. Transplantation: Retrospective data from Johns Hopkins University in Baltimore, emphasized that advanced age is no longer a barrier to successful outcomes with nonmyeloablative (NMA) haploidentical transplantation.

16. Chimeric

antigen receptor (CAR)-modified T cells targeting CD19: Recent advances in T-cell engineering have enabled clinical trials to evaluate the potential for adoptive transfer of T cells to target malignancy. All of the following statements about treatment(s) with engineered gene-modified T cells are true EXCEPT: a. T cells engineered to express a CAR can effectively target CD19 (CTL019) in patients with advanced treatmentrefractory chronic lymphocytic leukemia and pediatric acute lymphocytic leukemia. b. CTL019 therapy has a potential role in DLBCL and primary mediastinal large B-cell lymphoma. c. Adoptive transfer of CTL019 cells engineered to express CD137 and T-cell receptor (TCR) zeta-chain signaling domains cannot result in expansion in vivo. d. CTL019 therapy is associated with a significant cytokine release syndrome that responds rapidly to anticytokine treatment.

anemia—stratified patients into favorable and unfavorable risk categories. In patients with no or one risk factor, the three-year OS was 59% (95% CI, 44%74%) and in patients with two or more risk factors the three-year OS was 9% (95% CI, 1%-18%). The database analysis used patients treated between 1993 and 2007, before approval of brentuximab. Böll B, Goergen H, Arndt N, et al. Relapsed Hodgkin lymphoma in older patients: a comprehensive analysis from the German Hodgkin Study Group. J Clin Oncol. 2013;31:4431-4437, PMID: 24190119.


CURRENT PRACTICE

CLINICAL ONCOLOGY NEWS • JANUARY 2014 • CLINICALONCOLOGY.COM

4. False. In this retrospective anal-

ysis the median OS rate from the time of transplantation for patients attaining sCR was not reached compared with patients achieving conventional CR (n=37; OS, 81 months) or near CR (nCR; n=91; OS, 60 months; P<0.001). Fiveyear OS rates were 80%, 53% and 47% for sCR, CR and nCR, respectively. Kapoor P, Kumar SK, Dispenzieri A, et al. Importance of achieving stringent complete response after autologous stem-cell transplantation in multiple myeloma. J Clin Oncol. 2013 Nov 18. [Epub ahead of print], PMID: 24248686.

5. True. This Phase II international

trial is the first to demonstrate the clinical activity of PD-1 blockade in DLBCL. Given these results, PD-1 blockade after auto-HSCT using pidilizumab (CT-011, CureTech) may represent a promising therapeutic strategy in this disease. Armand P, Nagler A, Weller EA, et al. Disabling immune tolerance by programmed death-1 blockade with pidilizumab after autologous hematopoietic stem-cell transplantation for diffuse large B-cell lymphoma: results of an international phase II trial. J Clin Oncol. 2013;31:4199-4206, PMID:24127452.

6. True. The first study was a retro-

spective review of 1,230 patients with AML in first CR who underwent alloHCT between 2000 and 2006. Multivariate analysis showed significantly less nonrelapse mortality (relative risk [RR], 0.58; 95% CI, 0.39-0.86; P=0.007), and relapse after, but not before, oneyear post-transplant (RR, 0.23; 95% CI, 0.08-0.65; P=0.006), and better leukemia-free survival (RR, 0.70; 95% CI, 0.55-0.88; P=0.003) and survival (RR, 0.68; 95% CI, 0.52-0.88; P=.003) in individuals receiving IV, but not oral, Bu compared with TBI. The second study was a nonrandomized prospective study of 1,483 patients with myeloid malignancies treated between 2009 and 2011. The only concern of note in the latter study was a higher incidence of hepatic veno-occlusive disease for Bu (5%) compared with TBI (1%). There were no differences in PFS and GVHD. This study also compared patients receiving Bu-fludarabine and Bu-cyclophosphamide; there was no difference in outcomes with these preparative regimens. Copelan EA, Hamilton BK, Avalos B, et al. Better leukemia-free and overall survival in AML in first remission following cyclophosphamide in combination with busulfan compared with TBI. Blood. 2013;122:3863-3870, PMID: 24065243. Bredeson C, LeRademacher J, Kato K, et al. Prospective cohort study comparing intravenous busulfan to total body irradiation in hematopoietic cell transplantation. Blood. 2013;122:3871-3878, PMID: 24081656. Champlin RE. Busulfan or TBI: answer to an age-old question. Blood. 2013;122:3856-3857, PMID:24311716.

7.

False. In this retrospective review of 388 patients with PV, ET, post ETPF, post PV-PF and PMF, investigators noted that bone marrow testing is not

required to detect JAK2 p.V617F mutation or quantify JAK2 p.V617F allele burden, because peripheral blood testing is highly reliable. Hence, a negative result of the JAK2 p.V617F mutation in a peripheral blood sample can be considered to be reliable, and obtaining a bone marrow sample for further testing does not seem to be necessary. They also caution readers that the sensitivity of the assay used in the current study and setting is 5%. Takahashi K, Patel KP, Kantarjian H, et al. JAK2 p.V617F detection and allele burden measurement in peripheral blood and bone marrow aspirates in patients with myeloproliferative neoplasms. Blood. 2013;122:3784-3786, PMID: 24068492.

8. True.

Sixteen patients with a median of 6.5 prior therapies (including auto-HSCT [n=15], auto- and alloSCT [n=4]) were enrolled in dose escalation (n=1) or recommended phase 2 dose (n=15) cohorts. At the median follow-up of 3.2 months, 15 (94%) patients demonstrated decreased fluorodeoxyglucose uptake including two CRs (13%) and five partial responses (PRs; 31%) using CHESON criteria, with a total CR+PR rate of 44%. Enrollment continues. Janku F, Garrido-Laguna I, Velez-Bravo V, et al. Significant activity of the mTOR inhibitor sirolimus and HDAC inhibitor vorinostat in heavily pretreated refractory Hodgkin lymphoma patients. ASH Annual Meeting Abstracts. Blood. 2013;122:3048.

9. False. These results were very

encouraging for patients with MZL. The percentage of two-year PFS was 71%. Of the 38 patients (18 small lymphocytic lymphoma, 12 lymphoplasmacytic lymphoma and eight MZL), seven achieved a complete remission and 14 a partial remission, with an overall response rate (ORR) of 55%. Seven patients had stable disease and five had lymphoma progression. In general, the R2 regimen was relatively well tolerated. Patients received oral rituximab 20 mg once daily on days 1 to 21. Lenalidomide was administered at a dose of 375 mg/m2 on day 14 of every course. Treatment was repeated every 28 days for up to six cycles. Sacchi S, Pozzi S, Cesaretti M, et al. Final results of a phase II study of lenalidomide in combination with rituximab for the treatment of indolent nonfollicular, non-Hodgkin lymphoma. ASH Annual Meeting Abstracts. Blood. 2013;122:4383.

10. True.

This interim analysis shows that four cycles of RiBVD (plan for six cycles in patients with responsive disease) are very effective, with an ORR of 87% and a CR/unconfirmed CR of 60% in untreated older adult patients (n=70) with MCL. Four patients (6%) died: one from pneumonia, two from cardiac arrest and one following progressive multifocal leukoencephalopathy. To date, three patients have progressed after three cycles. Gressin R, Callanan M, Daguindau N, et al. The Ribvd regimen (rituximab IV, bendamustine

IV, velcade SC, dexamethasone IV) offers a high complete response rate in elderly patients with untreated mantle cell lymphoma. Preliminary results of the Lysa trial “Lymphome Du Manteau 2010 SA.” ASH Annual Meeting Abstracts. Blood. 2013;122:370.

11. False. The combination of ibruti-

nib and R-CHOP has an acceptable safety profile in treatment-naive patients with NHL, with no new toxicities noted. The recommended phase II dose was established as 560 mg ibrutinib. A separate randomized study (ClinicalTrials. gov: NCT01855750) is comparing the efficacy and safety of ibrutinib in combination with R-CHOP with R-CHOP alone in adult patients with newly diagnosed non-germinal center DLBCL. Younes A, Flinn I, Berdeja J, et al. Combining ibrutinib with rituximab, cyclophosphamide, doxorubicin, vincristine, and prednisone (R-CHOP): updated results from a Phase 1b study in treatment-naïve patients with CD20-positive B-cell non-Hodgkin’s lymphoma (NHL). ASH Annual Meeting Abstracts. Blood. 2013;122:852.

12.

True. The effect was particularly pronounced in men with low International Prognostic Index (IPI) score (IPI <1; EFS at three years, 91.2% vs. 78.2%), whereas this benefit was not as strong among men with IPI greater than 1 (77.1% vs. 70.2%) or women with IPI less than 1 (79.5% vs. 82.5%) or greater than 1 (74.7% vs. 75.7%). Maintenance therapy may be compensating for underdosing in current rituximab-containing induction regimens among men, indicating that rituximab dosing for men with aggressive B-cell lymphoma should be prolonged or increased, particularly in the low IPI subgroup. Jaeger U, Trneny M, Melzer H, et al. Rituximab maintenance significantly prolongs event free (EFS) and progression free survival (PFS) in male patients with aggressive B-cell lymphoma in the NHL13 study. ASH Annual Meeting Abstracts. Blood. 2013;122:851.

13.

True. A retrospectively defined analysis considering only EFS events from the time when treatment was different between the two arms showed a statistically significant increase in EFS with long-term maintenance compared with the shorter eight-month maintenance regimen. Long-term rituximab maintenance also doubled the median PFS without leading to increased undue toxicity. Taverna CJ, Martinelli G, Hitz F, et al. Rituximab maintenance treatment for a maximum of 5 years in follicular lymphoma: results of the randomized Phase III trial SAKK 35/03. ASH Annual Meeting Abstracts. Blood. 2013;122:508.

14. True. Relapse of FL after firstline treatment with R-CHOP is associated with poor outcomes. However, progression of disease within two years of R-CHOP defines a unique category of patients at substantially increased risk for death. In an unselected cohort of 588 patients, 63% (65 of 104) of deaths

within six years of follow-up occurred in patients who progressed within two years. The aforementioned four risk factors conferred a significantly high risk for early progressive disease. According to the investigators, these high-risk patients represent a distinct population that warrants further study in directed prospective studies of FL biology and clinical trials. Casulo C, Byrtek M, Dawson KL, et al. Early relapse of follicular lymphoma after R-CHOP uniquely defines patients at high risk for death: an analysis from the National Lymphocare Study. ASH Annual Meeting Abstracts. Blood. 2013;122:510.

15.

True. The investigators retrospectively analyzed the outcomes of 273 consecutive patients with poor-risk or advanced hematologic malignancies, aged 50 to 75 years, who received NMArelated haploidentical transplantation with post-transplant cyclophosphamide. Eligibility included ECOG performance score of 1 or less or 2; left ventricular ejection fraction no less than 35%; adequate pulmonary and renal function; and no uncontrolled infection. Fortyone patients (15%) had prior auto-HSCT and 138 (51%) had a high-risk comorbidity index score (HCT-CI ≥3). The twoyear PFS probabilities for patients in their 50s, 60s and 70s were 39%, 36% and 39%, respectively ((P=0.9), with corresponding two-year OS probabilities of 51%, 56% and 44% ((P=0.9). Kasamon YL, Prince G, Bolaños-Meade J, et al. Encouraging outcomes in older patients (pts) following non-myeloablative (NMA) haploidentical blood or marrow transplantation (haploBMT) with high-dose posttransplantation cyclophosphamide (PT/Cy). ASH Annual Meeting Abstracts. Blood. 2013;122:158.

16.

C. Adoptive transfer of CTL019 cells engineered to express CD137 and TCR zeta-chain signaling domains can result in in vivo expansion, homing to marrow, and long-term functional persistence of engineered cells, accompanied by ongoing complete clinical responses and long-term B cell aplasia in a substantial fraction of patients with advanced, refractory and high-risk B-cell malignancies. Kalos M, Nazimuddin F, Finklestein JM, et al. Long-term functional persistence, B cell aplasia and anti-leukemia efficacy in refractory B cell malignancies following T cell immunotherapy using CAR-redirected T cells targeting CD19. ASH Annual Meeting Abstracts. Blood. 2013;122:163. Kochenderfer JN, Dudley ME, Kassim SH, et al. Effective treatment of chemotherapy-refractory diffuse large B-cell lymphoma with autologous T cells genetically engineered to express an anti-CD19 chimeric antigen receptor. ASH Annual Meeting Abstracts. Blood. 2013;122:168. Porter DL, Kalos M, Frey NV, et al. Chimeric antigen receptor modified T cells directed against CD19 (CTL019 cells) have long-term persistence and induce durable responses in relapsed, refractory CLL. ASH Annual Meeting Abstracts. Blood. 2013;122:4162.

19


Visit

ClinicalOncology.com Late-breaking akin k News Links to Other Other Therapeutic Areas Fr CME Free

Medical Educa Education tion Archives i Most Popular Articles A rticles & R Reader Comments Tabbed Navigatiion ion n

Opti mize d for wide scr disp een lays Educational ucational Reviews R i


JANUARY 2014 Brought to You by

REPORT Addressing Misconceptions About Selective Internal Radiation Therapy Williams Medical Center in Provhe development of liver idence, Rhode Island, explained metastases in patients with Faculty that although surgical resection metastatic colorectal cancer is the gold standard in the man(mCRC) presents unique chalN. Joseph Espat, MD, MS, FACS agement of these tumors,2 sevlenges for oncologists and surProfessor of Surgery geons working to formulate an eral factors that may limit who is Boston University School of Medicine effective treatment strategy. Liver eligible for such therapy should Boston, Massachusetts metastases have been shown be considered. “I make a deciChief, Surgical Oncology to occur in 50% of patients with sion on whether or not a patient Roger Williams Medical Center CRC,1 and a majority of these is resectable based on how Providence, Rhode Island much liver I will have left over, patients present with metastases whether resection of the tumor is that cannot be resected.2 Thus, Michael J. Pishvaian, MD, PhD going to involve removal of vital it is important that clinicians colAssistant Professor vascular structures, or whether laborate to use various treatment Department of Hematology/Oncology the patient’s tumor is in a potenoptions that allow for resection Lombardi Comprehensive Cancer Center tially difficult location. Other conin some cases and improve outsiderations, including patient comes overall.2 The 5-year overGeorgetown University Medical Center comorbidities and general perall survival (OS) rate for patients Washington, DC formance status, are also taken with mCRC remains low at 12% into account,� he said. despite improvements for all The standard treatment paradigm for patients with patients with CRC over the past several decades and metastatic disease includes systemic chemotheradespite the development of several new classes of agents.3 py with unresectable metastatic disease and targeted Reports in the United States estimated that 142,000 new biologic therapies,5 although each of these modalities cases of CRC were diagnosed in 2013, with approximately 50,000 deaths anticipated to occur among all diagnosed. has either limited efficacy or is associated with sysAs a result, treatments for mCRC must continue to evolve.4 temic adverse events (AEs),5,6 and most patients with late-stage mCRC ultimately become refractory to these N. Joseph Espat, MD, MS, FACS, professor of surtherapies.7 To enhance the treatment armamentarium for gery at Boston University School of Medicine in Boston, Massachusetts, and chief of Surgical Oncology at Roger patients with metastatic disease, specifically those with

T

Supported by


REPORT A

B

C

Figure 1. SIR-Spheres microspheres mode of action. A) SIR-Spheres microspheres are administered into the hepatic arteries where they travel through the bloodstream to the tumors. B) Around 3 times the diameter of red blood cells, but with a similar specific gravity, SIR-Spheres microspheres are carried by the blood flow directly to the tumor vasculature. C) The microspheres kill cancer cells by delivering beta radiation directly to the tumor, while sparing normal liver tissue from damage. Based on reference 11. Images courtesy of Sirtex Medical Inc.

liver metastases, the concept of liver-directed therapy was developed.8 This treatment approach harnesses the fact that liver metastases derive their blood supply from the hepatic arteries, whereas normal liver tissue derives most of its blood supply from the portal vein. Liver-directed therapy involves directly infusing treatments into the hepatic arteries and/or starving the metastases of their arterial blood supply.7

Selective Internal Radiation Therapy

“As a result, delivery of agents and devices via the hepatic artery has the potential to selectively target the liver tumor without targeting the normal tissue.” Therefore, when infused via a catheter placed into the hepatic artery, these microspheres travel to the tumor and lodge primarily in the tumor microvasculature, thereby inducing tumor cell death through high-dose interstitial radiotherapy (Figure 1).11 This special report addresses common misconceptions about the use of SIRT and SIR-Spheres microspheres to help guide physicians toward optimal management of patients with mCRC and liver metastases.

One such liver-directed therapy, selective internal radiation therapy (SIRT), delivers radiation to liver tumors in a locoregional fashion, thereby preventing the radiation exposure to normal liver parenchyma or other organs that would occur SIR-Spheres Microspheres: with external beam radiation.9,10 Yttrium-90 (Y-90), a highEfficacy and Safety Data energy, beta-emitting isotope, is used most often with SIRT due to its controlled depth of tissue penetration and ease of Dr. Pishvaian noted that there is a misconception that use.9 SIR-Spheres microspheres (Sirtex Medical Inc.) are the SIR-Spheres microspheres lack prospective efficacy and safety data (Table 1).13-18 “There are a number of prospective only FDA PMA-approved* microspheres for the treatment of 11 liver metastases from CRC. SIR-Spheres microspheres conclinical trials that support the safety and efficacy of SIR-Spheres microspheres for the management of liver metassist of biocompatible polymer microspheres that average 32.5 tases from colorectal canmicrons in diameter.11,12 cer,” he said. “The safety Michael J. Pishvaian, MD, profile is favorable, and in PhD, assistant professor in “...Delivery of agents and devices via the hepatic several preliminary studies, the Division of Hematology/ artery has the potential to selectively target the liver we see a survival rate that Oncology at the Lombarappears to be better than di Comprehensive Cancer tumor without targeting the normal tissue.” when systemic chemotherCenter at Georgetown Uni—Michael J. Pishvaian, MD, PhD apy alone is used.” versity in Washington, DC, For example, Hendlisz explained the principles by et al performed a prospecwhich this agent can act in tive, multicenter, Phase III trial that randomized 46 patients a selective manner: “Selective or focused treatment of the liver with nonresectable, chemotherapy-refractory, liver-limited capitalizes on the difference in anatomic perfusion when commCRC to 5-fluorouracil (5-FU) with or without SIR-Spheres paring the normal liver parenchyma and tumors within the liver. microspheres therapy.13 Median time to tumor progression While the normal liver tissue is fed primarily through the portal venous system, the angiogenic factors secreted by liver tumors (2.1 vs 4.5 months) and median OS (7.3 vs 10 months) favored result in formation of neovessels that supply the liver tumor— the SIR-Spheres arm.13 In another prospective, multicenter these neovessels arise off of the hepatic arterial system,” he said. study, van Hazel et al performed a randomized Phase II trial of * Full Indication: SIR-Spheres microspheres are indicated for the treatment of unresectable metastatic liver tumors from primary colorectal cancer with adjuvant intra-hepatic artery chemotherapy (IHAC) of FUDR (Floxuridine).

2


REPORT with a minimal side-effect profile, leading the authors to state SIR-Spheres microspheres plus 5-FU versus 5-FU alone in that the use of both modalities shows promising efficacy for patients with advanced CRC.14 These authors reported that some mCRC patients.16 response rate and time to disease progression (18.6 vs 3.6 months; PFS also was shown to be favorP<0.0005) were significantly higher able when Sharma et al evaluat“...There have been a number of in the SIR-Spheres microspheres ed SIR-Spheres microspheres in plus 5-FU group than in the 5-FU combination with FOLFOX4 (oxalistudies that have demonstrated the monotherapy group.14 Furtherplatin, leucovorin, and 5-FU) as firstsafety and efficacy of Y-90 microline therapy. Twenty patients with more, median survival was signifspheres when used concurrently CRC and nonresectable liver metasicantly longer for patients receiving tases experienced an average PFS the combination treatment than for with chemotherapy...” of 9.3 months (range, 3.9-30.7 those receiving 5-FU monotheramonths), which the authors deemed py (29.4 vs 12.8 months; P=0.02).14 —N. Joseph Espat, MD, MS, FACS favorable.17 This study provided Other prospective studies have shown similar results. In an evalproof-of-concept information and uation of SIR-Spheres microformed the basis for the Phase III spheres therapy, 50 patients with liver metastases from SIRFLOX study, which is discussed later in this report. CRC (the majority of whom had received at least 4 prior lines of chemotherapy) underwent treatment with SIR-Spheres.15 Using SIRT and SIR-Spheres Microspheres Median OS was 12.6 months and the 2-year survival rate was First-Line Therapy 19.6%, which was deemed favorable in these heavily pretreat15 ed patients. A later second-line study by van Hazel et al aimed Another fallacy related to the use of SIRT is “the misconcepto assess the benefit of using SIR-Spheres microspheres plus tion that SIRT can only be used in the salvage setting, which concomitant systemic chemotherapy in 5-FU– refractory mCRC probably stems from the fact that although the FDA registration patients.16 Results showed patients receiving SIR-Spheres study was conducted in mainly first-line patients, the majority of the early institutional experiences were in this salvage setmicrospheres plus irinotecan had median progression-free ting,” said Dr. Pishvaian. Dr. Espat agreed that clinicians might survival (PFS) rates of 6 months and OS rates of 12 months

Table 1. Results From Selected Clinical Trials of SIR-Spheres Microspheres Investigator

n

Treatment

ORR, %

TTP/PFS, mo

Survival, mo

Kosmider et al, 2011

12 7

SIR-Spheresa + FOLFOX SIR-Spheresa + 5-FU/LV

Not applicable

10.4

29.4

Sharma et al, 2007

20

SIR-Spheresa + FOLFOX4

90

9.3

Not reported

van Hazel et al, 2004

11 10

SIR-Spheresa + 5-FU/LV 5-FU/LV

91 0 (P<0.001)

18.6 3.6 (P<0.0005)

29.4 12.8 (P=0.02)

25

Irinotecan plus SIR-Spheresa

48

6

12.2

Cosimelli et al, 2010

50

SIR-Spheresa

24

3.7

12.6

Hendlisz et al, 2010

21

SIR-Spheresa + protracted 5-FU Protracted 5-FU

10 0 (P=0.22)

4.5 2.1 (P=0.03)

10 7.3 (P=0.8)

FIRST-LINE

SECOND-LINE van Hazel et al, 2009

SALVAGE THERAPY

23

5-FU, 5-fluorouracil; FOLFOX4, oxaliplatin, leucovorin, 5-FU; LV, leucovorin; ORR, overall response rate; PFS, progression-free survival; TTP, time to progression a

SIR-Spheres microspheres.

Adapted from references 13-18. SIR-Spheres® is a registered trademark of Sirtex SIR-Spheres Pty Ltd.

3


REPORT Table 2. SIRFLOX 120 Patient Safety Data: Constitutional Adverse Events Constitutional Treatment-Related Adverse Events

Oxaliplatin Package Insert Safety Data (n=259)

SIRFLOX Study Patient Group (n=122)

Grade 1-4, %

Grade 3-4, %

Grade 1-4, %

Grade 3-4, %

Nausea

71

6

59

2

Vomiting

41

4

31

3

Diarrhea

69

12

47

7

Constipation

32

4

22

0

Abdominal pain

29

8

18

3

Adapted from reference 22.

not be considering beyond salvage therapy for the use of SIRthat is assessing the effect of adding Y-90 microspheres Spheres microspheres. “Since those early investigations, there to standard chemotherapy (mFOLFOX6 ± bevacizumab) in have been a number of studies18,19 that have demonstrated the patients with nonresectable liver metastases from primary colorectal cancer who have not received prior chemothersafety and efficacy of Y-90 microspheres when used concurapy,” he said. “We already have favorable preliminary saferently with chemotherapy in earlier-line settings,” said Dr. Espat. ty data from this study, in which the adverse event profile In the FDA registration study for SIR-Spheres microof Y-90 microspheres therspheres,20 Gray et al reviewed apy is comparable or betthe efficacy of a single infusion ter than that associated with of SIR-Spheres microspheres “The use of Y-90 microspheres increases the components of standard plus floxuridine administered proportion of patients who become eligible for chemotherapy regimens.” via hepatic arterial pump resection of their liver metastases...” Interim safety data from the versus floxuridine alone in SIRFLOX study are outlined 70 patients with nonresect—N. Joseph Espat, MD, MS, FACS in Table 2. 22 The SIRFLOX able liver metastases, 85% of whom had not received prior study completed enrolment systemic chemotherapy for in April 2013 and data for the their colorectal liver metastases. Results from this pivotal trial primary end point, PFS, are expected in 2014. showed patients who received SIR-Spheres microspheres with In the United Kingdom, the randomized Phase III FOXFIRE floxuridine had improved survival rates compared with patients clinical trial—a large, first-line study similar to SIRFLOX— who received chemotherapy alone, and SIR-Spheres microis investigating the use of SIR-Spheres microspheres as therspheres more than doubled the tumor response rate as well as apy in patients with liver-only or liver-predominant mCRC. the time to disease progression.19 This study, with an OS primary end point, will be combined with FOXFIREGlobal, an ex-UK extension of the study to enroll Additionally, Kosmider et al reported early experience additional patients over 3 years.23 and follow-up data on the use of SIR-Spheres microspheres along with either FOLFOX or 5-FU as first-line treatment in SIRT and Surgical Resection 19 patients with liver-dominant mCRC.18 Results showed that use of SIRT plus standard first-line therapy provided complete Evaluation for possible resection or ablation of liver metasresponse for 2 patients (11%), partial response for 14 patients tases following treatment with SIR-Spheres microspheres (74%), and stable disease for 1 patient (5%); median PFS was with or without chemotherapy is vital, as resection is the gold 10.4 months. Median OS for all patients was 29.4 months and standard treatment for mCRC in the liver. 2 Although hepat9 patients remained alive after a median follow-up period ic resection previously was reserved for patients who satisof 18.6 months (range, 3.2-78.5 months). Overall, among fied specific criteria (eg, a maximum of 3 lesions in the same patients who experienced AEs, most events were low grade.18 lobe if it was possible to achieve 1-cm margins; absence of portal lymph node metastases), advancements in both surgiBeyond currently available data, Dr. Pishvaian added that cal techniques and systemic therapy mean that these criteria upcoming study results should further document the utility may no longer apply. 24 In fact, improvements in the outcomes of SIRT and SIR-Spheres microspheres early in treatment. “The best data for the use of Y-90 microspheres in the earof patients with mCRC over the past 15 years have been lier-line setting will come from the ongoing SIRFLOX study. attributed to increased rates of hepatic resection in approThis is a large, multicenter, randomized controlled study 21 priate patients and advances in systemic chemotherapy and

4


REPORT biologic agents. As a result, the criteria for defining which patients are suited for surgical therapy have evolved, and A C many surgeons take an aggressive stance in the management of hepatic metastases.24,25 There is a misconception among many health care professionals that surgical resection is not possible, or is unsafe, after SIRT. In fact, “the use of Y-90 microspheres increases the proportion of patients who become eligible for resection of their liver metastases, through multiple mechanisms. First, its use is associated with rapid tumor volume reduction,” B D Dr. Espat said. Although only 10% to 20% of patients with mCRC and liver metastases present as resectable,1 the use of neoadjuvant chemotherapy to cytoreduce tumors can result in another 10% to 15% of patients being eligible for resection. 26 Cosimelli et al reported that following SIR-Spheres microspheres therapy, 4% of their patients, all of whom were heavily pretreated, experienced reductions in the volume of their liver metastases sufficient to enable potentially curative resection of 3 or more segments (Figure 2).15 Similarly, HenFigure 2. CRC liver metastasis dlisz et al reported that the tumor for one patient treated with CT scans. SIR-Spheres microspheres and 5-FU was downsized suffiContrast-enhanced pre-radioembolization arterial (A) ciently to allow right hepatectomy.13 and portal-venous-phase (B) CT scans showing large CRC liver metastasis. Six-month post-radioembolization “Second, similar to portal vein embolization, the use of (C) including significant attenuation, sharp margins and Y-90 microspheres is associated with liver hypertrophy,” added thin peripheral enhancement compatible with complete Dr. Espat. In patients with a limited liver function reserve, conlesion necrosis; confirmed by post-resection evaluation tralateral liver lobe hypertrophy after SIRT may be able to (D), showing fibrotic capsule (arrow). increase the chance for a successful liver resection, potenCRC, colorectal cancer; CT, computed tomography tially due to the increase in liver size.27 Ahmadzadehfar et al Reprinted by permission from Macmillan Publishers Ltd reported that SIRT was associated with a median increase in on behalf of Cancer Research UK: British Journal of contralateral liver lobe volume of 30% (P= 0.001) in patients Cancer. Cosimelli M, Golfieri R, Cagol PP, et al; Italian Society of Locoregional Therapies in Oncology (SITILO). with metastases in both liver lobes, and a 70% increase Multi-centre phase II clinical trial of yttrium-90 resin (P= 0.01) in patients with metastases only in the treated liver microspheres alone in unresectable, chemotherapy lobe. 27 These investigators concluded that the contralaterrefractory colorectal liver metastases. Br J Cancer. al hypertrophy induced by SIRT might allow subsequent liver 2010;103(3):324-331, copyright 2010. resection in a larger proportion of patients than would otherwise be eligible for such therapy.27 Dr. Pishvaian added, “Although there have been no trials that explicitly studied resection after Y-90 microspheres therIn a prospective study of 840 patients with primary or metapy, you can see that there are reports within the prospecastatic lesions undergoing hepatectomy, 40 patients undertive studies of patients went preoperative who have successfulhepatic arterial ther“SIR-Spheres microspheres are small enough ly undergone resecapy with either Y-90 tion after such therapy. or drug-eluting bead with a similar density to red blood cells that they... At Georgetown Medi( DEB ) chemotherapy, are carried by the bloodstream all the way to the cal Center, we have had 160 patients had predirect experience with operative systemic chetumor vasculature.” patients who received motherapy, and 640 Y-90 microspheres therpatients had no preoper—Michael J. Pishvaian, MD, PhD apy and subsequentative treatment. Results ly underwent hepatic from this analysis found resection without any unexpected side effects or increases in that there was no observable difference in perihepattoxicity.” Other clinical evaluations have found similar safety ic inflammation or adhesions, and the degree of hepatresults when patients who received Y-90 microsphere theraic inflammation in non-tumor–bearing liver was similar py underwent surgery: Whitney et al reported their experifor all 3 groups (with at most, moderate inflammation in ence in 15 patients with CRC that had spread to the liver, with patients receiving preoperative hepatic arterial therapy). 29 one patient having no specific hepatic complications followThe authors concluded that the use of preoperative hepatic ing hepatectomy (80% tumor necrosis) after treatment with arterial therapy before hepatectomy in well-selected patients SIR-Spheres microspheres.28 is safe and shows similar morbidity and mortality when compared with patients receiving preoperative systemic chemoStudy data also have shown that Y-90 microspheres therapy or no preoperative treatment. 29 therapy does not increase complication risk after surgery.

5


REPORT Table 3. Comparison of Selected Intra-Arterial Therapies Attribute

SIR-Spheres Microspheres

TACE/DEBs

Particle size

24-38 microns

100-700 microns

Mode of action

Internal tumor irradiation

Vessel occlusion with systemic delivery of chemotherapy to tumor and parenchyma

Embolic effect

Minimal vessel occlusion

Significant vessel occlusion

Procedure length

Outpatient 4-6 h

Inpatient 4-6 d

Subsequent LDT use

No limitations

Significant limitations

Most common side effects

Grade 1-2 fatigue, fever for up to 12 d

Grade 2-3 nausea, pain, fatigue for up to 6 wk

FDA status

FDA approved for mCRC

Not FDA-approved

mCRC level 1 evidence

Yes

No

DEBs, drug-eluting beads; LDT, liver-directed therapy; mCRC, metastatic colorectal cancer; TACE, transarterial chemoembolization Data on file. From reference 31.

SIRT Treatment and Chemotherapy

significantly longer in patients with or without extrahepatic metastases when SIRT was added to 5-FU than when cheSurgical intervention and chemotherapy have increased motherapy alone was used.14 The ongoing SIRFLOX study long-term remission for patients with liver metastases, a 2 patient population that typically has a very poor prognosis. also is expected to provide further information about treatment of patients with extrahepatic metastases. SIRFLOX has Although the addition of Y-90 microspheres therapy may proenrolled patients with and without extrahepatic metastases to vide added benefits, patient considerations related to surgery receive SIR-Spheres microspheres therapy.30 and chemotherapy should be reviewed beforehand. “Of note, the sequencing of therapy is very important in order to use Y-90 microspheres in an effective manner,” Dr. Espat said. Intra-Arterial Therapy: Differing Modalities “If the patient has received ablation therapy or even chemotherapy in the last 30 to 60 days, particularly an antiMisconceptions related to the mode of action of SIRT are angiogenic therapy, the target tumor may not be sufficiently also prevalent. Dr. Pishvaian reviewed the use of intra-artevascularized to enable optimal delivrial therapy: “Intra-arterial antineoplasery of the Y-90 therapy to the tumor.” tic therapy has evolved considerably SIR-Spheres microspheres However, initiating chemotherapy and over the past 2 decades. The first form scheduling SIRT when the patient of intra-arterial therapy was to simtherapy is available at more presents in a specific condition (eg, ply occlude those vessels thought to than 700 centers worldwide when disease is confined to the liver preferentially feed tumors,” he said. or if the patient develops extrahepat“Later, chemotherapy was added to with at least 300 centers in ic disease) may prove less benefithis therapy [eg, transarterial chethe United States alone. cial than using SIRT alongside other moembolization (TACE)]. However, options early in the treatment course, both bland embolization and TACE according to Dr. Pishvaian. can result in postembolization syn“Although Y-90 microsphere therapy only treats the liver drome, an expected side effect of TACE, which consists metastases, the proven efficacy and safety of SIRT when of nausea, vomiting, abdominal pain, and fever, and can given concurrently with chemotherapy means that the treatrequire ongoing hospitalization, supportive care, and monment can be used safely in patients who also have extraheitoring. The next refinement was the development of DEBs patic metastases,” Dr. Pishvaian said. “If a patient has bulky that can be infused into the relevant arteries in an attempt disease in the liver and has extrahepatic disease, you could to deliver locally high doses of chemotherapy to the tumor.” safely give Y-90 microspheres therapy to control the liver Unfortunately, many of these patients still develop postembodisease while simultaneously giving systemic chemotheralization syndrome and often require hospital care (Table 3).31 py to control the extrahepatic disease.” In one of the afore“In contrast, SIR-Spheres microspheres are small mentioned studies by van Hazel et al, median survival was enough with a similar density to red blood cells that they

6


REPORT regional liver-directed therapy in patients with liver metasdo not cause an embolic effect because they are carried tases caused by CRC that are considered nonresectable,11 by the bloodstream all the way to the tumor vasculature,” Dr. Pishvaian said. “The nature and its utility has been valiof the radiation emitted by dated by postmarketing surthese microspheres means that veillance with in excess of More than 37,000 doses have been the radiation only travels about 37,000 patients treated at more 2 mm, meaning that exposure than 700 centers worldwide.33 supplied globally as of mid-2013. of normal tissues to the radiPhysicians should considation is minimal. As a result, er use of Y-90 microspheres in these patients do not develop patients who are unfit for tumor postembolization syndrome, and the overwhelming majoriresection or when potential tumor reduction could lead to ty of patients can be treated on an ambulatory basis without future resection. Data have shown that surgery following an overnight hospital stay.” use of Y-90 microspheres plus chemotherapy is safe and its Dr. Espat raised another misconception with regard to use concurrently with systemic chemotherapy can help treat intra-arterial therapies, which is that their clinical effectivepatients with extrahepatic disease. Specifically, several studness is similar. “Clinicians should be aware that transarteriies have shown that SIR-Spheres microspheres in combinaal chemoembolization and DEBs tend to be less effective for tion with chemotherapy provide better rates of survival than colorectal metastases than for primary hepatocellular carcichemotherapy alone. The majority of data supporting the use noma. This is because hepatocellular carcinomas are highly of SIR-Spheres microspheres is in the salvage therapy setvascular, whereas the colorectal metastases tend to be vasting, but the ongoing SIRFLOX study is expected to characcularized only in their periphery. In less vascular tumors, the terize the utility of this agent as a first-line treatment for mCRC larger DEBs are limited by physical properties (ie, size and patients with liver metastases. 21 density) from delivering their payload to the tumor. In contrast, the physical and radiation-emitting properties of Y-90 microReferences spheres still allow for local radiation delivery to the metastases, despite only peripheral vascularization.” 1. Van den Eynde M, Hendlisz A. Treatment of colorectal liver metas-

SIRT Reimbursement and Availability According to the manufacturer, most insurance companies generally cover the cost of SIR-Spheres microspheres for the treatment of mCRC. Under the terms of the Medicare Prescription Drug, Improvement, and Modernization Act of 2003, Medicare, for the most part, reimburses hospitals for the full cost of outpatient treatment with SIR-Spheres microspheres.32 Many private payors also acknowledge the clinical utility of the SIRT procedure and have issued positive coverage policies for treatment. These payors include Aetna, Anthem, Cigna, Health Net, Humana, UnitedHealthcare, WellPoint, and many independent Blue Cross Blue Shield plans, as well as many other smaller plans on a case-by-case basis.32 “For the indication of colorectal metastasis to the liver, it has not been difficult to get funding from most insurance carriers over the last few years,” Dr. Pishvaian noted. In terms of advice for their colleagues working in areas or institutions in which this therapy is not offered, Drs. Pishvaian and Espat recommended that physicians consult the manufacturer’s website or contact the manufacturer directly to determine their closest referral center. According to the manufacturer, SIR-Spheres microspheres therapy is available at more than 700 centers worldwide with at least 300 centers in the United States alone. More than 37,000 doses have been supplied globally as of mid-2013.33

Conclusion Liver metastasis from CRC is common and is associated with poor prognosis.2 A significant proportion of patients with liver metastases are either not candidates for curative surgical resection or become refractory to chemotherapy and/or targeted agents. 2,7 SIR-Spheres microspheres is the only FDA-approved Y-90 microsphere indicated for

tases: a review. Rev Recent Clin Trials. 2009;4(1):56-62. 2. Adam R. Chemotherapy and surgery: new perspectives on the treatment of unresectable liver metastases. Ann Oncol. 2003;14(suppl 2):ii13-ii16. 3. National Cancer Institute. SEER Cancer Statistics Review 19752010. SEER Stat Fact Sheets: Colon and Rectum. http://seer. cancer.gov/statfacts/html/colorect.html#survival. Accessed December 20, 2013. 4. American Cancer Society. Cancer Facts & Figures. www.cancer. org/acs/groups/content/@epidemiologysurveilance/documents/ document/acspc-036845.pdf. Accessed December 20, 2013. 5. Luu C, Arrington AK, Schoellhammer HF, et al. Targeted therapies in colorectal cancer: surgical considerations. J Gastrointest Oncol. 2013;4(3):328-336. 6. Tol J, Koopman M, Rodenburg CJ, et al. A randomised phase III study on capecitabine, oxaliplatin and bevacizumab with or without cetuximab in first-line advanced colorectal cancer, the CAIRO2 study of the Dutch Colorectal Cancer Group (DCCG). An interim analysis of toxicity. Ann Oncol. 2008;19(4):734-738. 7. Cyjon A, Neuman-Levin M, Rakowsky E, et al. Liver metastases from colorectal cancer: regional intra-arterial treatment following failure of systemic chemotherapy. Br J Cancer. 2001;85(4): 504-508. 8. Ye LC, Liu TS, Ren L, et al. Randomized controlled trial of cetuximab plus chemotherapy for patients with KRAS wild-type unresectable colorectal liver-limited metastases. J Clin Oncol. 2013;31(16):1931-1938. 9. Wang SC, Bester L, Burnes JP, et al. Clinical care and technical recommendations for 90yttrium microsphere treatment of liver cancer. J Med Imaging Radiat Oncol. 2010;54(3):178-187. 10. Stubbs RS, Wickremesekera SK. Selective internal radiation therapy (SIRT): a new modality for treating patients with colorectal liver metastases. HPB (Oxford). 2004;6(3):133-139.

7


REPORT 11. SIR-Spheres (yttrium-90) microspheres [package insert]. Woburn, MA: Sirtex Medical Inc.; 2011. 12. Data on file. Sirtex Medical Inc. 13. Hendlisz A, Van den Eynde M, Peeters M, et al. Phase III trial comparing protracted intravenous fluorouracil infusion alone or with yttrium-90 resin microspheres radioembolization for liver-limited metastatic colorectal cancer refractory to standard chemotherapy. J Clin Oncol. 2010;28(23):3687-3694. 14. Van Hazel G, Blackwell A, Anderson J, et al. Randomised phase 2 trial of SIR-Spheres plus fluorouracil/leucovorin chemotherapy versus fluorouracil/leucovorin chemotherapy alone in advanced colorectal cancer. J Surg Oncol. 2004;88(2):78-85. 15. Cosimelli M, Golfieri R, Cagol PP, et al; Italian Society of Locoregional Therapies in Oncology (SITILO). Multi-centre phase II clinical trial of yttrium-90 resin microspheres alone in unresectable, chemotherapy refractory colorectal liver metastases. Br J Cancer. 2010;103(3):324-331. 16. van Hazel GA, Pavlakis N, Goldstein D, et al. Treatment of fluorouracil-refractory patients with liver metastases from colorectal cancer by using yttrium-90 resin microspheres plus concomitant systemic irinotecan chemotherapy. J Clin Oncol. 2009;27(25):4089-4095. 17. Sharma RA, Van Hazel GA, Morgan B, et al. Radioembolization of liver metastases from colorectal cancer using yttrium-90 microspheres with concomitant systemic oxaliplatin, fluorouracil, and leucovorin chemotherapy. J Clin Oncol. 2007;25(9):1099-1106. 18. Kosmider S, Tan TH, Yip D, et al. Radioembolization in combination with systemic chemotherapy as first-line therapy for liver metastases from colorectal cancer. J Vasc Interv Radiol. 2011;22(6):780-786. 19. Gray B, Van Hazel G, Hope M, et al. Randomised trial of SIRSpheres plus chemotherapy vs. chemotherapy alone for treating patients with liver metastases from primary large bowel cancer. Ann Oncol. 2001;12(12):1711-1720. 20. Food and Drug Administration. SIR-SpheresÂŽ - P990065. Summary of Safety and Effectiveness Data. http://www.accessdata.fda. gov/cdrh_docs/pdf/P990065b.pdf. Accessed December 20, 2013. 21. ClinicalTrials.gov. FOLFOX Plus SIR-SPHERES MICROSPHERES Versus FOLFOX Alone in Patients With Liver Mets From Primary

Colorectal Cancer (SIRFLOX). http://clinicaltrials.gov/ct2/show/NCT 00724503?term=sirflox&rank=1. Accessed December 20, 2013. 22. Sirtex Medical Inc. Y-90 outcomes in colorectal oncology. www. slideshare.net/SirtexMedical/y90-outcomes-in-colorectal-oncology. Accessed December 20, 2013. 23. Sharma RA, Wasan HS, Love SB, et al; FOXFIRE Trial Management Group. FOXFIRE: a phase III clinical trial of chemo-radio-embolisation as first-line treatment of liver metastases in patients with colorectal cancer. Clin Oncol (R Coll Radiol). 2008;20(3):261-263. 24. Khatri VP, Petrelli NJ, Belghiti J. Extending the frontiers of surgical therapy for hepatic colorectal metastases: is there a limit? J Clin Oncol. 2005;23(33):8490-8499. 25. Kopetz S, Chang GJ, Overman MJ, et al. Improved survival in metastatic colorectal cancer is associated with adoption of hepatic resection and improved chemotherapy. J Clin Oncol. 2009;27(22):3677-3683. 26. Adam R, Delvart V, Pascal G, et al. Rescue surgery for unresectable colorectal liver metastases downstaged by chemotherapy: a model to predict long-term survival. Ann Surg. 2004;240(4):644657; discussion 657-668. 27. Ahmadzadehfar H, Meyer C, Ezziddin S, et al. Hepatic volume changes induced by radioembolization with 90Y resin microspheres. A single-centre study. Eur J Nucl Med Mol Imaging. 2013;40(1):80-90. 28. Whitney R, Tatum C, Hahl M, et al. Safety of hepatic resection in metastatic disease to the liver after yttrium-90 therapy. J Surg Res. 2011;166(2):236-240. 29. Brown RE, Bower MR, Metzger TL, et al. Hepatectomy after hepatic arterial therapy with either yttrium-90 or drug-eluting bead chemotherapy: is it safe? HPB (Oxford). 2011;13(2):91-95. 30. SIRFLOX. Study design. www.sirflox.com/study-design. Accessed December 20, 2013. 31. Data on file. Sirtex Medical Inc. 32. Sirtex Medical Inc. SIR-Spheres microspheres coding sheet. http:// sirtex.com/media/61003/2013_sirtex_coding_sheet_final_v_673u-0213.pdf. Accessed December 20, 2013. 33. Data on file. Sirtex Medical Inc.

Disclosures: Dr. Espat reported no relevant financial conflicts of interest. Dr. Pishvaian reported receiving speaker fees from Sirtex Medical Inc.

Copyright Š 2014, McMahon Publishing, 545 West 45th Street, New York, NY 10036. Printed in the USA. All rights reserved, including the right of reproduction, in whole or in part, in any form. 809-U-1213

8

SR1327

Disclaimer: This monograph is designed to be a summary of information. While it is detailed, it is not an exhaustive clinical review. McMahon Publishing, Sirtex Medical Inc., and the authors neither affirm nor deny the accuracy of the information contained herein. No liability will be assumed for the use of this monograph, and the absence of typographical errors is not guaranteed. Readers are strongly urged to consult any relevant primary literature.


Turn static files into dynamic content formats.

Create a flipbook
Issuu converts static files into: digital portfolios, online yearbooks, online catalogs, digital photo albums and more. Sign up and create your flipbook.